Что такое функции, области определения и значений функции
Определение 1
Функция — вид зависимости, при котором каждому элементу одного множества ставится в соответствие элемент другого множества.
В общем виде функцию в алгебре обозначают как y=f(x). Переменную x называют независимой переменной или аргументом функции, переменную y — зависимой переменной или значением функции.
Основными характеристиками функции являются:
область определения;
область значений.
Определение 2
Область определения — множество значений, которые может принимать аргумент функции, то есть переменная x. Область определения иногда называют областью допустимых значений. Обозначение области допустимых значений функции f: D (f).
Также область определения можно трактовать как проекцию графика функции на ось абсцисс.
Определение 3
Областью значений называется множество всех значений функции (переменной y), полученных при переборе всех значений переменной x из области определения. Принято следующее обозначение области значений: E (f).
В графическом изображении область значений — проекция графика функции на ось ординат.
Нахождение области значений осуществляется одним из следующих способов:
графически;
аналитически (по уравнению).
Способы нахождения области значений некоторых функций по графику
Чаще всего графический способ используют для функций с достаточно простой зависимостью. В этом случае построение графика не вызывает трудностей.
Приведем алгоритм нахождения области значений функции по графику:
Ищем область определения функции. Например, у показательной функции или параболы аргумент может принимать любое значение из множества действительных чисел R, то есть E(f)=R. Если выражение f(x) является дробным, область определения находится из условия неравенства нулю знаменателя. Если выражение f(x) находится под квадратным корнем, область определения можно узнать из неравенства f(x)≥0.
Строим график функции по точкам.
По графику функции находим ее минимум. Значение y_{min} будет являться нижней границей области значений. В том случае, когда минимум невозможно определить визуально, то есть функция не имеет минимума, границей будет -∞.
Аналогично определяем максимум y_{max} и, соответственно, верхнюю границу области значений. Если максимум не определяется, границей области значения является +∞.
Записываем область значений функции, при этом необходимо учесть точки разрыва, если они есть. Точки разрыва возникают, например, при исключении из области определения таких значений аргументов, при которых знаменатель обращается в ноль. Область значений записывают в виде числового промежутка. Границы, входящие в область, заключают в квадратные скобки, не входящие — в круглые. Если область значений включает в себя несколько числовых промежутков, их объединяют знаком «U», например: (-∞; 4]U[6; +∞).
Как найти область значений функции по уравнению
Нахождение области значений функции по заданному уравнению также сводится к вычислению экстремумов.
Рассмотрим два случая:
Нахождение области значений функции, непрерывной на некотором заданном отрезке.
Нахождение области значений функции, непрерывной на некотором интервале. Сюда же отнесем случаи, когда функция не существует в какой-либо точке. Например, точка нуля знаменателя, в которой функция не существует, а область определения терпит разрыв.
Алгоритм поиска области значений для первого случая:
Находим производную функции.
Приравниваем производную к нулю, находим корни уравнения f′(x)=0 и точки, в которых производная не существует — критические точки.
Отмечаем корни, критические точки и границы заданного интервала на прямой и определяем знаки производной на каждом получившемся промежутке.
Находим минимумы и максимумы функции. Если в некоторой точке x1 производная меняет знак с «+» на «-», то точка x1 — максимум, если с «-» на «+» — минимум.
Подставляя значения аргументов для минимума и максимума функции в выражение f(x), находим минимальное и максимальное значения функции. В том случае, если имеются точки, в которых производная не существует, значение функции вычисляем через пределы по формулам: limx→x1-0f(x) и limx→x1+0f(x).
Записываем область значений функции.
Для второго случая:
Находим производную, приравниваем ее к нулю и определяем знаки производной на каждом промежутке.
Определяем значение функции в каждой из точек. Для определения значения функции в граничных точках, а также в точках разрыва или точках, в которых производная не существует, вычисляем пределы функции аналогично указанным в пункте 5 для первого случая.
Определяем и записываем область значений.
Примеры решений
Рассмотрим несколько примеров на нахождение области значений функции и приведем их решения.
Задача 1
Найти область значений функции y=x по графику.
Решение:
Найдем область определения функции. Выражение под знаком квадратного корня всегда положительно, то есть x≥0, и область определения D(f(x))=[0; +∞). Теперь построим график функции.
Из графика видно, что минимальное значение переменная y принимает при x=0. Максимальное значение не определяется, при этом видно, что при возрастании x значении y также растет. Получили, что ymin=0, а область значений E(f(x))=[0; +∞).
Ответ: E(f(x))=[0; +∞).
Задача 2
Найти область значений функции y=4xx2+2 на отрезке [-2; 2].
Решение:
Найдем область определения функции. Функция представляет собой дробь, однако, ее знаменатель не будет равен нулю при любых значениях x. Действительно, квадрат любого числа есть положительное число, получили в знаменателе сумму положительных чисел. Тогда D=R, где R — множество действительных чисел.
Приравняем числитель производной к нулю и найдем корни получившегося уравнения: 8-4×2=0;x1=-2иx2=2.
Отметим корни на координатной оси и, поочередно подставляя значения x=-4,-2,2,4, определим знаки производной на каждом промежутке.
Из рисунка видно, что функция имеет один минимум и максимум. Вычислим значения ymin и ymax:
ymin=y(-2)=4·(-2)(-2)2+2=-2;
ymax=y(2)=4·(2)(2)2+2=2.
Экстремумы функции входят в заданный интервал и не являются точками разрыва области определения функции, то есть минимальные и максимальные значения должны быть включены в область значений.
Ответ:E(f(x))=[-2;2].
Задача 3
Найти область значений функции y=5x+1 на области действительных чисел.
Решение:
Найдем область определения функции. Знаменатель не может быть равным нулю, значит, D(f(x))=(-∞; -1)U(-1;+∞).
Найдем производную: y'(x)=-5(x+1)2.
Получили, что производная не равна нулю при любых x. При x=-1 знаменатель производной обращается в ноль, то есть в данной точке производная не существует.
Отметим точку x=-1 и рассмотрим два промежутка: (-∞;-1) и (-1;+∞).
Определим знаки производной на каждом промежутке.
Из рисунка видно, что функция убывает на обоих интервалах и не имеет максимума или минимума.
Теперь определим значение функции в точке x=-1, для чего вычислим пределы функции при x→-1-0 и x→-1+0.
limx→(-1-0)5x+1=5-1-0+1=5-0=-∞;
limx→(-1+0)5x+1=5-1+0+1=5+0=+∞.
Итак, точка x=-1 — это точка разрыва второго рода.
Значение функции на границах заданного интервала -∞ и +∞ также вычисляется с помощью пределов:
limx→-∞5x+1=5-∞+1=0;
limx→+∞5x+1=5+∞+1=0.
Данная функция является гиперболой с асимптотами x=-1 и y=0.
Область значений E(f(x))=(-∞; 0)U(0;+∞).
Ответ: E(f(x))=(-∞; 0)U(0;+∞).
Область блока, область действия и локальная область видимости в Javascript
Я не уверен, что вы уже получили ответы на свои вопросы:
Область блока иногда совпадает с областью действия функции? Я знаю, что область действия функции предназначена для всего внутри функции, но не понимаю, что
точно область блока.
Да , область действия блока иногда совпадает с областью действия функции. Область блока — это все, что находится внутри набора фигурных скобок { здесь область блока } . Таким образом, в верхней части кода функции область блока будет такой же, как и область действия функции:
функциональный тест(х) {
// это и область видимости блока, и область действия функции
пусть у = 5;
если (х) {
// это меньшая область видимости блока, которая не совпадает с областью действия функции
пусть г = 1;
}
}
Для Javascript в настоящее время рекомендуется использовать вместо этого let / const
var для будущего обслуживания? (Это было из Руководства по стилю Airbnb)
let и const являются частью новейшей спецификации ES6 и реализованы только в последних движках Javascript, а иногда в последних движках они включаются только со специальными флагами. Они появляются во всех новых движках/браузерах JS, но пока не получили широкого распространения. Таким образом, если вы пишете Javascript для обычного использования браузера в широком Интернете, вы не можете надежно использовать пусть и пока не равны .
В некоторых случаях вы можете безопасно программировать с let и const now:
nodejs или плагин только для конкретной версии конкретного браузера).
Если вы используете транспилятор, который преобразует ваш код в код, который будет работать во всех браузерах. При использовании транспилятора вы можете написать свой код с использованием новейших функций, а транспилятор «заглушит» его, чтобы ваш код работал в старых браузерах, используя симуляции новых функций.
Если вы программируете для среды, где вы знаете, что поддерживаются let и const , рекомендуется использовать их соответствующим образом. Если вы объявите переменную в начале своей функции, то let и var сделают то же самое.
Если вы объявите переменную в меньшей области внутри функции, то пусть будет содержаться в меньшей области, но var будет поднята наверх функции и будет иметь область действия, независимо от того, где она объявляется.
Руководство по стилю AirBnb, на которое вы ссылаетесь, специально написано для среды ES6 (обратите внимание, что существует отдельная ссылка на версию их руководства по стилю для ES5). Итак, это означает, что они предполагают среду с поддержкой ES6. Это либо потому, что они нацелены на серверный JS-движок, который, как они знают, поддерживает ES6, либо потому, что они используют транспилятор, который преобразует код ES6 во что-то, что будет работать на движке ES5.
Примечание о транспиляторах. Прежде чем использовать транспилятор и переключать все объявления переменных на пусть в рамках блоков, стоит понять, какой код генерирует транспилятор и влияет ли генерируемый им дополнительный код на производительность вашего приложения. Например, область блока для let моделируется путем создания встроенного IIFE, что может привести к дополнительным накладным расходам во время выполнения для каждого блока, содержащего оператор let .
Окружность — геометрическая фигура, состоящая из всех точек плоскости, расположенных на заданном расстоянии от данной точки.
Данная точка (O) называется центром окружности. Радиус окружности — это отрезок, соединяющий центр с какой-либо точкой окружности. Все радиусы имеют одну и ту же длину (по определению). Хорда — отрезок, соединяющий две точки окружности. Хорда, проходящая через центр окружности, называется диаметром. Центр окружности является серединой любого диаметра. Любые две точки окружности делят ее на две части. Каждая из этих частей называется дугой окружности. Дуга называется полуокружностью, если отрезок, соединяющий её концы, является диаметром. Длина единичной полуокружности обозначается через π. Сумма градусных мер двух дуг окружности с общими концами равна 360º. Часть плоскости, ограниченная окружностью, называется кругом. Круговой сектор — часть круга, ограниченная дугой и двумя радиусами, соединяющими концы дуги с центром круга. Дуга, которая ограничивает сектор, называется дугой сектора. Две окружности, имеющие общий центр, называются концентрическими. Две окружности, пересекающиеся под прямым углом, называются ортогональными.
Взаимное расположение прямой и окружности
Если расстояние от центра окружности до прямой меньше радиуса окружности (d ), то прямая и окружность имеют две общие точки. В этом случае прямая называется секущей по отношению к окружности.
Если расстояние от центра окружности до прямой равно радиусу окружности, то прямая и окружность имеют только одну общую точку. Такая прямая называется касательной к окружности, а их общая точка называется точкой касания прямой и окружности.
Если расстояние от центра окружности до прямой больше радиуса окружности, то прямая и окружность не имеют общих точек
.
Центральные и вписанные углы
Центральный угол — это угол с вершиной в центре окружности. Вписанный угол — угол, вершина которого лежит на окружности, а стороны пересекают окружность.
Теорема о вписанном угле
Вписанный угол измеряется половиной дуги, на которую он опирается.
Следствие 1. Вписанные углы, опирающиеся на одну и ту же дугу, равны.
Следствие 2. Вписанный угол, опирающийся на полуокружность — прямой.
Теорема о произведении отрезков пересекающихся хорд.
Если две хорды окружности пересекаются, то произведение отрезков одной хорды равно произведению отрезков другой хорды.
Основные формулы
Длина окружности:
C = 2∙π∙R
Длина дуги окружности:
R = С/(2∙π) = D/2
Диаметр:
D = C/π = 2∙R
Длина дуги окружности:
l = (π∙R) / 180∙α, где α — градусная мера длины дуги окружности)
Площадь круга:
S = π∙R2
Площадь кругового сектора:
S = ((π∙R2) / 360)∙α
Уравнение окружности
В прямоугольной системе координат уравнение окружности радиуса r с центром в точке C (xо;yо) имеет вид:
(x — xо)2 + (y — yо)2 = r2
Уравнение окружности радиуса r с центром в начале координат имеет вид:
x2 + y2 = r2
Площадь круга
Другие заметки по алгебре и геометрии
Полезная информация?
Взаимное расположение прямой и окружности – онлайн-тренажер для подготовки к ЕНТ, итоговой аттестации и ВОУД
Запомнить
Восстановить пароль
Регистрация
Конспект
Существует 3 случая взаимного расположения прямой и окружности в зависимости от соотношения между радиусом r окружности и расстоянием d прямой от центра окружности.
1. d < r. Если расстояние от центра окружности до прямой меньше радиуса окружности, то окружность и прямая имеют две общие точки.
2. d = r. Если расстояние от центра окружности до прямой равно радиусу окружности, то прямая и окружность имеют единственную общую точку.
3. d > r. Если расстояние от центра окружности до прямой больше радиуса окружности, то прямая и окружность не имеют общих точек.
Прямая, имеющая с окружностью ровно одну общую точку, называется касательной к окружности, а их общая точка называется точкой касания прямой и окружности. Прямая, имеющая с окружностью две общие точки, называется секущей.
Теоремы о касательных и секущих
Касательная к окружности перпендикулярна радиусу, проведенному к точке касания.
Если из данной точки проведены к окружности две касательные, то отрезки касательных равны между собой и центр окружности лежит на биссектрисе угла с вершиной в этой точке: \(AB=AC\). 2=CD\cdot BC\).
Произведение всего отрезка одной секущей на его внешнюю часть равно произведению всего отрезка другой секущей на его внешнюю часть: \(AC\cdot BC=EC\cdot DC\).
Сообщить об ошибке
Обязательные
Математическая грамотность
Грамотность чтения
История Казахстана
Предметы по профилю
Биология
Химия
Английский язык
Французский язык
География
Немецкий язык
Информатика
Основы права
Русская литература
Математика
Физика
Русский язык
Всемирная история
Укажите предмет *
Скопируйте и вставьте вопрос задания *
Опишите подробнее найденную ошибку в задании *
Прикрепите скриншот
Объем файла не должен превышать 1МБ
Казахский
Русский
Обратите внимание! По выбранным Вами предметам ГРАНТЫ не предоставлены. В AlmaU, Университете Нархоз и Каспийском Университете представлены специальности, где профильными предметами являются математика, физика, география, иностранный язык, Человек. Общество. Право, всемирная история, биология, химия и творческий экзамен.
1. Скачайте приложение iTest, используя QR-код или строку поиска в AppStore или Play Market
2. Авторизуйтесь в приложении и готовьтесь к экзаменам вместе с нами
Круг и линия
СОЗДАНО ВДОХНОВЛЯТЬ
АРТ-ОБЪЕКТЫ НА КАЖДЫЙ ДЕНЬ
КУПИТЬ >
Материал Править
Затем каждый компонент методично полируется и покрывается матовым лаком для достижения приятного и долговечного результата. Индивидуальные вариации в каждом изделии добавляют глубину и размер, чтобы подчеркнуть их уникальный характер ручной работы.
КУПИТЬ СЕЙЧАС
Представляем наш новый
Купить сейчас template—147404
390__featured_collection_1.title»>
КОЛЛЕКЦИЯ КАРЬЕРА
Коллекция Quarry вдохновлена природными органическими формами: от Rill and Clast, напоминающего гладкие изношенные края речной скалы, до волнообразных движений настенных ковров Ripple. Латунные формы нагреваются и формируются вручную для создания трехмерных композиций.
КУПИТЬ СЕЙЧАС >
Настенные ковры Ripple — отожженная латунь
Обычная цена
375,00 долларов США
Обычная цена
Цена продажи
375,00 долларов США
Цена за единицу товара
/чел.
Настенные ковры Ripple — латунь
Обычная цена
375,00 долларов США
Обычная цена
Цена продажи
375,00 долларов США
Цена за единицу товара
/чел.
Настенные ковры Ripple — черная патина
Обычная цена
375,00 долларов США
Обычная цена
Цена продажи
375,00 долларов США
Цена за единицу товара
/чел.
Настенная подвеска Rill — черная патина
Обычная цена
375,00 долларов США
Обычная цена
Цена продажи
375,00 долларов США
Цена за единицу товара
/чел.
Подвеска из глины — черная патина
Обычная цена
375,00 долларов США
Обычная цена
Цена продажи
375,00 долларов США
Цена за единицу товара
/чел.
Настенный подвес Quarry — черная патина
Обычная цена
450,00 долларов США
Обычная цена
Цена продажи
450,00 долларов США
Цена за единицу товара
/чел.
Настенный подвес Element — черная патина
Обычная цена
$325.00
Обычная цена
Цена продажи
$325.00
Цена за единицу товара
/чел.
Минеральная настенная подвеска — черная патина
Обычная цена
230,00 долларов США
Обычная цена
Цена продажи
230,00 долларов США
Цена за единицу товара
/чел.
Внутри студии
Загляните за кулисы нашей студии в Остине. Вы сможете увидеть дизайнерские идеи и процессы, лежащие в основе коллекций, то, как мы делаем изделия, а также наши идеи по стилю для вашего дома.
КОЛЛЕКЦИЯ BALANCE >
КОЛЛЕКЦИЯ FOLD >
CONTINUUM >
Лучшее из Нью-Йорка | Полный круиз по Манхэттену
Лучшее в Нью-Йорке
2,5 ЧАСА
PIER 83, MIDTOWN
Единственный экскурсионный круиз, полностью огибающий Манхэттен. Посмотрите на все достопримечательности Нью-Йорка с сидений в помещении с регулируемой температурой или с нашей огромной открытой террасы.
Место на открытой террасе
Закуски и полный бар
Фотопауза у Статуи Свободы
Гид
Круизы с воскресенья, 16 апреля
Лучшее из Нью-Йорка
ОБЗОР КРУИЗА
Отправьтесь в единственный экскурсионный круиз в Нью-Йорке, который проходит по острову Манхэттен с контролируемой температурой , современные лодки.
Во время этого 2-часового и 30-минутного круиза вы пройдете по всем трем рекам Нью-Йорка, пройдете под 20 мостами и увидите более 130 самых знаковых достопримечательностей города, включая Manhattan Skyline , Little Island , Hudson Yards , Один мировой торговый центр , Бруклинский мост и Downtown Brooklyn , yan stadium . внимательно посмотрите на Статую Свободы и Остров Эллис !
Наши всемирно известные гиды расскажут историю Нью-Йорка, пока вы путешествуете по всему острову и наслаждаетесь лучшими достопримечательностями, которые может предложить Нью-Йорк!
Что вы увидите
Наш флагманский круиз с лучшими видами + все достопримечательности Нью-Йорка
Устройтесь поудобнее, расслабьтесь и насладитесь захватывающим видом на горизонт Нью-Йорка и Статую Свободы, пока наши отмеченные наградами гиды рассказывают о каждой детали во время вашего путешествия. маршрут.
Эмпайр Стейт Билдинг
Имея высоту 1250 футов (381 м), это здание было самым высоким зданием в мире с 1931 года до завершения строительства Всемирного торгового центра в 1972 году. автомобильные украшения и шпиль по образцу решетки радиатора.
High Line
Необыкновенный надземный парк длиной 1,45 мили (2,33 км), построенный на заброшенном участке бывшей железной дороги.
Статуя Свободы
Леди Свобода высотой 305 футов (93 м) приветствует посетителей с тех пор, как она была завещана Америке Францией в 1886 году. 1954.
Морской порт на Саут-Стрит
Морской порт на Саут-Стрит — исторический район Нью-Йорка. Он представляет собой одну из старейших архитектур в центре Манхэттена.
Бруклинский мост
Самый длинный висячий мост на момент открытия в 1883 году. Его башни высотой 276 футов (84 м) построены из известняка, гранита и цемента.
Манхэттенский мост
Манхэттенский мост, построенный в 1901 году, представляет собой висячий мост, соединяющий Нижний Манхэттен с центром Бруклина.
Штаб-квартира Организации Объединенных Наций
Спроектированная международной командой архитекторов, ООН представляет собой международную зону с собственной пожарной службой и почтовой службой.
Особняк Грейси
Этот дом в федеральном стиле, в настоящее время резиденция мэров Нью-Йорка, когда-то был загородным поместьем, кафе-мороженым и музеем.
Yankee Stadium
За свою более чем 85-летнюю историю этот «собор бейсбола» принял 6 581 домашний матч регулярного сезона Yankee.
Мост Джорджа Вашингтона
Самый загруженный подвесной мост в мире проходит через реку Гудзон и пропускает более 103 миллионов автомобилей в год.
Бортовые удобства
Наши ультрасовременные лодки оснащены практически всем необходимым для спокойного и комфортного круиза.
Непревзойденные виды Выберите место на открытой палубе или насладитесь видом из огромных панорамных окон в наших крытых каютах с регулируемой температурой.
Вы сказали… Невероятная открытая палуба Лучший способ увидеть Нью-Йорк и Леди Свободу — это «на свежем воздухе» — на свежем воздухе, под прохладным бризом и теплым солнцем. Это не лучше, чем это!
Внутреннее пространство с регулируемой температурой Жарко или холодно, дождливо или солнечно, у нас есть крытая палуба с регулируемой температурой и достаточным количеством сидячих мест для всех
Бортовое кафе *Свежеприготовленное меню сэндвичей, салатов и закусок можно приобрести.
Освежающие напитки * Наслаждайтесь полным баром, предлагающим горячие и холодные напитки, включая пиво, вино и коктейли.
Бесплатный Wi-Fi Не нужно ждать, чтобы загрузить свои любимые фотографии и селфи!
Мост Джорджа Вашингтона
Гости смотрят на мост Генри Гудзона
Спланируйте свой визит
Приготовьтесь, соберитесь, проложите маршрут до пирса 83
Что делать по прибытии
Если вы приобрели билеты Premier непосредственно у Circleline. com, пожалуйста, предупредите посла в красной рубашке, и он поможет вам пройти через систему безопасности, чтобы быстро занять свое место.
Если вы купили стандартные билеты на сайтах Circleline.com, Viator, Get Your Guide или Expedia, пройдите прямо к посадке у входа на наш пирс. Пожалуйста, держите сумку открытой и готовой к проверке нашими сотрудниками службы безопасности.
Если у вас есть карта CityPASS и вы заранее забронировали места на сайте citypass.circleline.com, обратитесь непосредственно к службе безопасности.
Если у вас есть другой пропуск, например NY Pass, Explorer Pass или Sightseeing Pass, или ваучер, пройдите в нашу кассу, чтобы забрать билеты и пройти к линии контроля безопасности.
Парковка у пирса 83
Если вы едете на пирсе 83, у нас есть парковка для вас!
Мы предлагаем удобную парковку в нескольких шагах от наших лодок. У нас есть дежурный по парковке и персонал службы безопасности, чтобы обеспечить безопасность вашего автомобиля, а также мы предлагаем крытую парковку.
Онлайн-калькулятор матриц с исключением Гаусса в блоге Kelly Holland
Онлайн-калькулятор матриц с исключением Гаусса . Веб-программа на этой странице использует метод исключения Гаусса для решения любой системы линейных уравнений вида. Веб-математический метод исключения Гаусса (или перевод:
Калькулятор исключения Гаусса — изображение предоставлено: calculate.vg
Сложение веб-матриц, умножение, инверсия, вычисление определителя и ранга, транспонирование, приведение к диагональному, треугольному виду, возведение в степень, разложение lu, qr. A — матрица коэффициентов, x — вектор неизвестных. Умножьте первую строку так.
Калькулятор исключения Гаусса
Веб-программа на этой странице использует метод исключения Гаусса для решения любой системы линейных уравнений вида. Веб-калькулятор исключения Гаусса введение в калькулятор исключения Гаусса Джордана калькулятор Гаусса — это бесплатный онлайн-инструмент, используемый для преобразования матрицы в уменьшенную. Это онлайн-инструмент алгебры. С помощью этого онлайн-калькулятора вы это сделаете.
кредит изображения: calculate.vg
Online Matrix Calculator Исключение Гаусса — Веб-программа на этой странице использует исключение Гаусса для решения любой системы линейных уравнений формы. Веб-калькулятор исключения Гаусса введение в калькулятор исключения Гаусса Джордана калькулятор Гаусса — это бесплатный онлайн-инструмент, используемый для преобразования матрицы в уменьшенную. Онлайн матричный калькулятор Исключение Гаусса.
изображение предоставлено: abisaikun.blogspot.com
Онлайн матричный калькулятор Исключение Гаусса — Сеть в линейной алгебре, исключение Гаусса (также известное как редукция строк) — это алгоритм решения систем линейных уравнений. Калькулятор Гаусса — это бесплатный онлайн-инструмент, используемый для преобразования матрицы в уменьшенную ступенчатую форму. Онлайн матричный калькулятор Исключение Гаусса.
изображение предоставлено: www.scribd.com
Онлайн-калькулятор матриц с исключением Гаусса — Красивый бесплатный матричный калькулятор от desmos.com. Если перед переменной в уравнении нет числа. Онлайн матричный калькулятор Исключение Гаусса.
кредит изображения: calculate-online.net
Онлайн-калькулятор матриц Исключение Гаусса — С помощью этого онлайн-калькулятора вы сможете. Красивый бесплатный матричный калькулятор от desmos.com. Онлайн матричный калькулятор Исключение Гаусса.
изображение предоставлено: www.pbhconsulto.co
Online Matrix Calculator Исключение Гаусса — Обычно понимается как последовательность операций. A — матрица коэффициентов, x — вектор неизвестных. Онлайн матричный калькулятор Исключение Гаусса.
Решение методом Крамера системы линейных уравнений 3-4-го порядка
Решать системы линейных алгебраических уравнений второго, третьего, изредка четвертого порядка методом Крамера достаточно часто придется студентам младших курсов учебы при изучении основ линейной алгебры. Для большинства студентов стационарной формы учебы такие задания не являются сложными, однако кто выбрал заочную учебу или дистанционную, или пропустил по определенным причинам практические занятия, вычисления выглядят непонятными и тяжелыми. Чтобы исправить такую ситуацию в данной статье будут приведены наиболее распространены примеры данной темы и схема их решения. Если Вы хорошо поймете принцип их решения, то на практике у Вас не будет трудностей с подобными заданиями.
Для начала выберем задание из сборника задач Дубовика В.П., Юрика І.І. «Высшая математика».
————————————
Примеры
Решить систему линейных алгебраических уравнений.
1) (1. 153)
2) (4. 165)
3) (4. 174)
Решение.
1) В случае двух уравнений решение можно получить более простым способом. Выражаемый из второго уравнения
и подставим в первое
Раскрыв скобки, сгруппируем подобные слагаемые
Отсюда получим решение
Переменнуюнайдем подстановкой в любое из уравнений
Таким образом решением системы двух уравнений будут следующие значения
Поскольку цель статьи научить студентов решать по методике Крамера то решим данный пример и етим методом.
Для этого выпишем систему линейных уравнений в виде
Найдем детерминант основной части
Для вычисления вспомогательных определителей ставим столбец свободных членов на место первой строки для и на место второй для . В результате получим
Подставим найденные значения в формулы Крамера
и найдем неизвестные
Из рассмотренного примера видим что вычисление при двух уравнениях с двумя неизвестными достаточно простые.
2) Запишем систему трех алгебраических уравнений в удобном для решения виде
Найдем детерминант системы по правилу треугольников
Для вычисления дополнительных определителей подставляем столбец свободных членов на место первого, второго и третьего столбцов. В результате получим
Вычисляем неизвестные за формулами Крамера
Для данного примера нахождения решения также не слишком сложно, хотя по сравнению с системой двух уравнений вычислений заметно прибавилось.
3) Записываем систему уравнений четвертого порядка в виде
Находим главный определитель системы. При вычислении детерминантов четвертого порядка их необходимо раскладывать за строками или столбцами у каторых больше всего нулей. Поскольку в данном случае нулей главный определитель не имеет то разложим его за первой строкой
и найдем соответствующие детермиінанты третьего порядка
Подставим найденные значения в определитель
По такой же схеме вычисляем вспомогательные определители, напомню лишь, что они образуются заменой столбца в главном определителе на столбец свободных членов (обозначен черным цветом).
Решающие системы с правилом Крамера — предварительное исчисление
Цели обучения
В этом разделе вы:
Оцените 2 × 2 детерминанты.
Используйте правило Крамера, чтобы решить систему уравнений с двумя переменными.
Оценить 3 × 3 определителей.
Используйте правило Крамера, чтобы решить систему из трех уравнений с тремя переменными.
Знать свойства определителей.
Мы научились решать системы уравнений с двумя переменными и тремя переменными, а также несколькими методами: подстановкой, сложением, методом исключения Гаусса, использованием обратной матрицы и построением графика. Некоторые из этих методов легче применять, чем другие, и они более подходят в определенных ситуациях. В этом разделе мы изучим еще две стратегии решения систем уравнений.
Вычисление определителя матрицы 2×2
Определитель — это действительное число, которое может быть очень полезным в математике, поскольку оно имеет множество применений, например, для вычисления площади, объема и других величин. Здесь мы будем использовать определители, чтобы выяснить, является ли матрица обратимой, используя элементы квадратной матрицы, чтобы определить, существует ли решение системы уравнений. Однако, возможно, одним из наиболее интересных приложений является их использование в криптографии. Защищенные сигналы или сообщения иногда отправляются закодированными в матрице. Данные могут быть расшифрованы только с помощью обратимой матрицы и определителя. Для наших целей мы сосредоточимся на определителе как признаке обратимости матрицы. Вычисление определителя матрицы включает в себя следование определенным шаблонам, описанным в этом разделе.
Найдите определитель матрицы 2 × 2
Определитель матрицы по данным
определяется как
Обратите внимание на изменение в обозначениях. Существует несколько способов указать определитель, в том числе и заменить скобки в матрице прямыми линиями,
Нахождение определителя матрицы 2 × 2
Найти определитель заданной матрицы.
Показать решение
Использование правила Крамера для решения системы двух уравнений с двумя переменными
Теперь мы познакомим вас с последним методом решения систем уравнений, использующим определители. Этот метод, известный как правило Крамера, восходит к середине 18 века и назван в честь его новатора, швейцарского математика Габриэля Крамера (1704-1752), который представил его в 1750 году во Введении к анализу линий алгебры. Правило Крамера — жизнеспособный и эффективный метод поиска решений систем с произвольным числом неизвестных при условии, что у нас есть такое же количество уравнений, как и неизвестных.
Правило Крамера даст нам единственное решение системы уравнений, если она существует. Однако, если система не имеет решения или имеет бесконечное число решений, на это будет указывать нулевой определитель. Чтобы выяснить, является ли система противоречивой или зависимой, придется использовать другой метод, такой как исключение.
Чтобы понять правило Крамера, давайте внимательно посмотрим, как мы решаем системы линейных уравнений, используя основные операции со строками. Рассмотрим систему двух уравнений с двумя переменными.
Мы исключаем одну переменную, используя операции со строками, и находим другую. Скажем, что мы хотим решить для Если уравнение (2) умножается на коэффициент, противоположный коэффициенту в уравнении (1), уравнение (1) умножается на коэффициент в уравнении (2), и мы складываем два уравнения, переменная будет удалена.
Теперь найдите
Аналогично, чтобы найти, мы исключим
Решение даст
Обратите внимание, что знаменатель для обоих и является определителем матрицы коэффициентов.
Мы можем использовать эти формулы для решения и , но правило Крамера также вводит новые обозначения:
определитель матрицы коэффициентов
определитель числителя в решении
определитель числителя в решении
Ключом к правилу Крамера является замена интересующего столбца переменных столбцом констант и вычисление определителей. Тогда мы можем выразить и как частное двух определителей.
Правило Крамера для систем 2×2
Правило Крамера — это метод, использующий детерминанты для решения систем уравнений, в которых число уравнений равно количеству переменных.
Рассмотрим систему двух линейных уравнений с двумя переменными.
Решение с использованием правила Крамера дается как
Если мы решаем для столбца, заменяемого постоянным столбцом. Если мы решаем, столбец заменяется постоянным столбцом.
Использование правила Крамера для решения системы 2 × 2
Решите следующую систему, используя правило Крамера.
Показать решение
Решить для
Найти
Решение:
Попробуйте
Используйте правило Крамера, чтобы решить систему уравнений 2 × 2.
Показать решение
Вычисление определителя матрицы 3 × 3
Найти определитель матрицы 2 × 2 несложно, но определить определитель матрицы 3 × 3 сложнее. Один из методов состоит в том, чтобы дополнить матрицу 3×3 повторением первых двух столбцов, получив матрицу 3×5. Затем вычисляем сумму произведений записей вниз по по каждой из трех диагоналей (сверху слева направо снизу) и вычтите произведения записей вверх по по каждой из трех диагоналей (слева снизу вверх справа). Это легче понять с визуальным и пример.
Найдите определитель матрицы 3×3.
Дополнить первыми двумя столбцами.
От верхнего левого угла к нижнему правому: умножьте числа по первой диагонали. Прибавьте результат к произведению записей по второй диагонали. Добавьте этот результат к произведению записей вниз по третьей диагонали.
Из нижнего левого угла в верхний правый: вычтите произведение записей вверх по первой диагонали. Из этого результата вычтите произведение вхождений вверх по второй диагонали. Из этого результата вычтите произведение вхождений вверх по третьей диагонали.
Алгебра выглядит следующим образом:
Нахождение определителя матрицы 3 × 3
Найдите определитель заданной матрицы 3 × 3
Показать решение
Дополните матрицу первыми двумя столбцами, а затем следуйте формуле . Таким образом,
Попробуйте
Найдите определитель матрицы 3 × 3.
Показать решение
Можно ли использовать тот же метод, чтобы найти определитель большей матрицы?
Нет, этот метод работает только для матриц и . Для больших матриц лучше всего использовать графическую утилиту или компьютерное программное обеспечение.
Использование правила Крамера для решения системы трех уравнений с тремя переменными
Теперь, когда мы можем найти определитель матрицы 3 × 3, мы можем применить правило Крамера для решения системы трех уравнений с тремя переменными. Правило Крамера является простым и следует шаблону, согласующемуся с правилом Крамера для матриц 2 × 2. Однако по мере увеличения порядка матрицы до 3 × 3 требуется гораздо больше вычислений.
Когда мы вычисляем, что определитель равен нулю, правило Крамера не указывает, имеет ли система решение или бесконечное число решений. Чтобы выяснить это, мы должны выполнить исключение в системе.
Рассмотрим систему уравнений 3 × 3.
где
Если мы записываем определитель, мы заменяем столбец постоянным столбцом. Если мы записываем определитель, мы заменяем столбец постоянным столбцом. Если мы записываем определитель, мы заменяем столбец постоянным столбцом. Всегда проверяйте ответ.
Решение системы 3 × 3 с помощью правила Крамера
Найдите решение данной системы 3 × 3 с помощью правила Крамера.
Показать решение
Использовать правило Крамера.
Затем
Решение:
Попробуйте
Используйте правило Крамера, чтобы решить матрицу 3 × 3.
Показать решение
Использование правила Крамера для решения несовместимой системы
Решите систему уравнений, используя правило Крамера.
Показать решение
Начнем с нахождения определителей
Мы знаем, что определитель, равный нулю, означает, что либо система не имеет решений, либо имеет бесконечное число решений. Чтобы увидеть, какой из них, мы используем процесс исключения. Наша цель — исключить одну из переменных.
Умножить уравнение (1) на
Добавить результат к уравнению
Получаем уравнение, которое неверно. Следовательно, система не имеет решений. График системы показывает две параллельные линии. См. (Рисунок).
Рис. 1.
Используйте правило Крамера для решения зависимой системы
Решите систему с бесконечным числом решений.
Показать решение
Сначала найдем определитель. Настройте матрицу, дополненную первыми двумя столбцами.
Затем
Поскольку определитель равен нулю, решения либо нет, либо существует бесконечное число решений. Мы должны выполнить исключение, чтобы узнать.
Умножьте уравнение (1) на и добавьте результат к уравнению (3):
Получение ответа на утверждение, которое всегда истинно, означает, что система имеет бесконечное число решений. Изобразив систему, мы видим, что две плоскости одинаковы и обе пересекают третью плоскость по прямой. См. (Рисунок). Рис. 2.
Понимание свойств определителей
Есть много свойств определителей. Здесь перечислены некоторые свойства, которые могут быть полезны при вычислении определителя матрицы.
Свойства определителей
Если матрица имеет верхнетреугольную форму, определитель равен произведению элементов по главной диагонали.
При перестановке двух строк определитель меняет знак.
Если две строки или два столбца идентичны, определитель равен нулю.
Если матрица содержит строку нулей или столбец нулей, определитель равен нулю.
Определитель обратной матрицы является обратной величиной определителя матрицы
Если какая-либо строка или столбец умножается на константу, определитель умножается на тот же коэффициент.
Иллюстрация свойств определителей
Проиллюстрируйте каждое из свойств определителей.
Покажите решение
Свойство 1 гласит, что если матрица имеет форму верхнего треугольника, определитель равен произведению элементов по главной диагонали.
Дополнить первыми двумя столбцами.
Затем
Свойство 2 указывает, что перестановка строк меняет знак. Учитывая
Свойство 3 гласит, что если две строки или два столбца идентичны, определитель равен нулю.
Свойство 4 гласит, что если строка или столбец равны нулю, определитель равен нулю. Таким образом,
Свойство 5 утверждает, что определитель обратной матрицы является обратной величиной определителя Таким образом,
Свойство 6 гласит, что если любую строку или столбец матрицы умножить на константу, определитель умножается на тот же коэффициент. Таким образом,
Использование правила Крамера и свойств определителя для решения системы
Найдите решение данной системы 3 × 3.
Показать решение
Используя правило Крамера, мы имеем
Обратите внимание, что второй и третий столбцы идентичны. Согласно свойству 3 определитель будет равен нулю, поэтому решения либо нет, либо решений бесконечное множество. Мы должны выполнить исключение, чтобы узнать.
Умножьте уравнение (3) на –2 и добавьте результат к уравнению (1).
Получение утверждения, являющегося противоречием, означает, что система не имеет решения.
Получите доступ к этим онлайн-ресурсам для получения дополнительных инструкций и практики с правилом Крамера.
Решение системы двух уравнений с помощью правила Крамера
Решение системы из трех уравнений с использованием правила Крамера
Ключевые понятия
Определитель для см. (рисунок).
Правило Крамера заменяет столбец переменной столбцом константы. Решения см. (рисунок).
Чтобы найти определитель матрицы 3×3, увеличьте первые два столбца. Сложите три диагональных элемента (слева вверху справа внизу) и вычтите три элемента по диагонали (слева внизу справа вверху). См. (Рисунок).
Чтобы решить систему из трех уравнений с тремя переменными с помощью правила Крамера, замените столбец переменных столбцом констант для каждого желаемого решения: см. (рисунок).
Правило Крамера также полезно для нахождения решения системы уравнений без решения или с бесконечным числом решений. См. (Рисунок) и (Рисунок).
Некоторые свойства определителей полезны при решении задач. Например:
Если матрица имеет форму верхнего треугольника, определитель равен произведению элементов по главной диагонали.
При перестановке двух строк определитель меняет знак.
Если две строки или два столбца идентичны, определитель равен нулю.
Если матрица содержит строку нулей или столбец нулей, определитель равен нулю.
Определитель обратной матрицы является обратной величиной определителя матрицы
Если какая-либо строка или столбец умножается на константу, определитель умножается на тот же коэффициент. См. (Рисунок) и (Рисунок).
Раздел Упражнения
Вербальные
1. Объясните, почему мы всегда можем вычислить определитель квадратной матрицы.
Показать решение
Определитель — это сумма и произведение элементов матрицы, поэтому вы всегда можете оценить это произведение, даже если оно в конечном итоге равно 0.
2. Изучая правило Крамера, объясните, почему нет единственного решения системы, когда определитель вашей матрицы равен 0. Для простоты используйте матрицу.
3. Объясните, что означает в терминах обратной матрицы наличие нулевого определителя.
Показать решение
Обратное не существует.
4. Определитель матрицы равен 3. Если поменять местами строки и умножить первую на 6, а вторую на 2, объясните, как найти определитель, и дайте ответ.
Алгебраический
Для следующих упражнений найдите определитель.
5.
Показать решение
6.
7.
Показать решение
8.
9.
Показать решение
10.
11.
Показать решение
12.
13.
Показать решение
14.
15.
Показать решение
16.
17.
Показать решение
18.
19.
Показать решение
20.
21.
Показать решение
22.
23.
Показать решение
24.
Для следующих упражнений решите систему линейных уравнений, используя правило Крамера.
25.
Показать решение
26.
27.
Показать решение
28.
29.
Показать решение
30.
31.
Показать решение
32.
33.
Показать решение
34.
Для следующих упражнений решите систему линейных уравнений, используя правило Крамера.
35.
Показать раствор
36.
37.
Показать решение
38.
39.
Показать решение
40.
41.
Показать решение
42.
43.
Показать решение
Бесконечное количество решений
44.
Технология
В следующих упражнениях используйте функцию определителя в графической утилите.
45.
Показать решение
46.
47.
Показать решение
48.
Реальные приложения
Для следующих упражнений создайте систему линейных уравнений для описания поведения. Затем вычислить определитель. Будет ли уникальное решение? Если да, то найти единственное решение.
49. Два числа в сумме дают 56. Одно число на 20 меньше другого.
Показать решение
Да; 18, 38
50. Два числа в сумме дают 104. Если сложить два раза первое число плюс два раза второе число, получится 208
51. Три числа в сумме дают 106. Первое число на 3 меньше первого второй номер. Третье число на 4 больше первого числа.
Показать решение
Да; 33, 36, 37
52. Три числа в сумме дают 216. Сумма первых двух чисел равна 112. Третье число на 8 меньше первых двух вместе взятых.
Для следующих упражнений создайте систему линейных уравнений для описания поведения. Затем решите систему для всех решений, используя правило Крамера.
53. Вы вкладываете 10 000 долларов США в два счета, на которые начисляются 8% и 5% годовых. В конце года на ваших объединенных счетах было 10 710 долларов. Сколько было вложено в каждый счет?
Показать решение
7000 долларов на первом счете, 3000 долларов на втором счете.
54. Вы инвестируете 80 000 долларов на два счета, 22 000 долларов на один счет и 58 000 долларов на другой счет. В конце года, при условии простых процентов, вы заработали 2470 долларов в виде процентов. Второй счет получает на полпроцента меньше, чем удвоенный процент по первому счету. Каковы процентные ставки для ваших счетов?
55. Кинотеатру необходимо знать, сколько билетов для взрослых и детей было продано из 1200 билетов. Если детский билет стоит $5,95, билеты для взрослых стоят 11,15 долларов США, а общая сумма выручки составила 12 756 долларов США, сколько было продано детских билетов и билетов для взрослых?
Show Solution
120 детей, 1080 взрослых
56. Концертный зал продает одиночные билеты по 40 долларов каждый и билеты для пар по 65 долларов. Если общий доход составил 18 090 долларов США и был продан 321 билет, то сколько было продано одиночных билетов и сколько билетов для пар?
57. Вы решили покрасить кухню в зеленый цвет. Вы создаете цвет краски, смешивая желтую и синюю краски. Вы не можете вспомнить, сколько галлонов каждого цвета вошло в вашу смесь, но вы знаете, что всего было 10 галлонов. Кроме того, вы сохранили квитанцию и знаете, что общая потраченная сумма составила 29 долларов США. .50. Если каждый галлон желтого цвета стоит 2,59 доллара, а каждый галлон синего стоит 3,19 доллара, сколько галлонов каждого цвета входит в вашу зеленую смесь?
Show Solution
4 галлона желтого цвета, 6 галлонов синего цвета
58. Вы продали два вида шарфов на фермерском рынке и хотели бы знать, какой из них более популярен. Всего было продано 56 шарфов, желтый шарф стоил 10 долларов, фиолетовый — 11 долларов. Если ваш общий доход составил 583 доллара, сколько желтых шарфов и сколько фиолетовых шарфов было продано?
59. В вашем саду росли помидоры двух видов: зеленые и красные. Красный весит 10 унций, а зеленый весит 4 унции. У вас есть 30 помидоров общим весом 13 фунтов 14 унций. Сколько у вас помидоров каждого вида?
Show Solution
13 зеленых помидоров, 17 красных помидоров
60. На рынке три самых популярных овоща составляют 53% продаж овощей. Продажи кукурузы на 4% выше, чем у брокколи, продажи которой на 5% выше, чем у лука. Какую долю рынка занимает каждый овощ?
61. На том же рынке три самых популярных фрукта составляют 37% от общего количества продаваемых фруктов. Клубники продают вдвое больше, чем апельсинов, а киви продают на один процент больше, чем апельсинов. Для каждого фрукта найдите процент от общего количества проданных фруктов.
Show Solution
Клубника 18%, апельсины 9%, киви 10%
62. Три группы выступили на концертной площадке. Первая группа взимала 15 долларов за билет, вторая группа взимала 45 долларов за билет, а последняя группа взимала 22 доллара за билет. Было продано 510 билетов на общую сумму 12 700 долларов. Если у первой группы было на 40 зрителей больше, чем у второй группы, сколько билетов было продано на каждую группу?
63. Кинотеатр продал билеты на три фильма. Билеты на первый фильм стоили 5 долларов, билеты на второй фильм — 11 долларов, а на третий фильм — 12 долларов. На первый фильм было продано 100 билетов. Общее количество проданных билетов составило 642, а общий доход составил 6 774 доллара. Сколько билетов на каждый фильм было продано?
Show Solution
100 для фильма 1, 230 для фильма 2, 312 для фильма 3
64. В прошлом году мужчины в возрасте 20–29, 30–39 и 40–49 лет составляли 78% заключенных в тюрьме. В этом году те же возрастные группы составили 82,08% населения. 20–29возрастная группа увеличилась на 20%, возрастная группа 30–39 лет увеличилась на 2%, а возрастная группа 40–49 лет уменьшилась до своего предыдущего населения. Первоначально в возрастной группе 30–39 лет заключенных было на 2% больше, чем в возрастной группе 20–29 лет. Определите процент заключенных для каждой возрастной группы в прошлом году.
65. В женской тюрьме по дороге общее число заключенных в возрасте от 20 до 49 лет составляло 5 525 человек. В этом году возрастная группа 20-29 лет увеличилась на 10%, возрастная группа 30-39 лет уменьшилась на 20%, а возрастная группа 40-49 летвозрастная группа удвоилась. Сейчас там 6040 заключенных. Первоначально в возрастной группе 30–39 лет было на 500 человек больше, чем в возрастной группе 20–29 лет. Определите количество заключенных для каждой возрастной группы в прошлом году.
Show Solution
20–29: 2 100, 30–39: 2 600, 40–49: 825
Для следующих упражнений используйте этот сценарий: Компания, заботящаяся о своем здоровье, решает приготовить пищевую смесь из миндаля, сушеной клюквы и орехов кешью в шоколаде. Информация о пищевой ценности этих продуктов показана на (Рисунок).
Жир (г)
Белок (г)
Углеводы (г)
Миндаль (10)
6
2
3
Клюква (10)
0,02
0
8
Кешью (10)
7
3,5
5,5
66. Для специальной «низкоуглеводной» трейловой смеси имеется 1000 штук смеси. Общее количество углеводов 425 г, общее количество жиров 570,2 г. Если орехов кешью на 200 штук больше, чем клюквы, сколько штук каждого предмета будет в смеси?
67. Для смеси «походной» в составе смеси 1000 шт., содержащих 390,8 г жира, 165 г белка. Если миндаля столько же, сколько орехов кешью, то сколько каждого элемента содержится в смеси?
Show Solution
300 миндаля, 400 клюквы, 300 кешью
68. Для смеси «Энергия-бустер» в смеси 1000 штук, содержащих 145 г белка и 625 г углеводов. Если количество миндаля и кешью в сумме равно количеству клюквы, сколько каждого элемента содержится в смеси?
Повторные упражнения
Системы линейных уравнений: две переменные
В следующих упражнениях определите, является ли упорядоченная пара решением системы уравнений.
1. и
Show Solution
№
2. и
В следующих упражнениях используйте замену для решения системы уравнений.
3.
Показать раствор
4.
5.
Показать решение
В следующих упражнениях используйте сложение для решения системы уравнений.
6.
7.
Показать решение
Решений не существует.
8.
Для следующих упражнений напишите систему уравнений для решения каждой задачи. Решите систему уравнений.
9. Фабрика имеет функцию затрат на производство и функцию дохода. Какова точка безубыточности?
Показать решение
10. Исполнитель взимает плату где — общее количество посетителей шоу. Заведение берет 75 долларов за билет. После того, как много людей купят билеты, место станет безубыточным, и какова общая стоимость билетов, проданных в этот момент?
Показать решение
Системы линейных уравнений: три переменные
В следующих упражнениях решите систему трех уравнений, используя подстановку или сложение.
11.
Показать решение
12.
13.
Показать решение
Решений не существует.
14.
15.
Показать решение
16.
17.
Показать решение
18.
Для следующих упражнений напишите систему уравнений для решения каждой задачи. Решите систему уравнений.
19. Три нечетных числа дают в сумме 61. Меньшее число на одну треть больше, а среднее число на 16 меньше большего. Какие три числа?
Показать решение
11, 17, 33
20. Билеты на спектакль в местном театре распроданы. Они продают все 500 билетов на общую сумму 8 070 долларов. Билеты стоили 15 долларов для студентов, 12 долларов для детей и 18 долларов для взрослых. Если группа продала в три раза больше билетов для взрослых, чем билетов для детей, сколько билетов каждого типа было продано?
Решение систем с помощью правила Крамера
Для следующих упражнений найдите определитель.
21.
Показать раствор
0
22.
23.
Показать решение
6
24.
В следующих упражнениях используйте правило Крамера для решения линейных систем уравнений.
25.
Показать решение
26.
27.
Показать решение
( х , 5 х + 3)
28.
29.
Показать решение
30.
Глоссарий
Правило Крамера
метод решения систем уравнений, имеющих такое же количество уравнений, что и переменных, с использованием определителей
определитель
число, рассчитанное с использованием элементов квадратной матрицы, определяющее такую информацию, как наличие решения системы уравнений
Использование правила Крамера для решения системы трех уравнений с тремя переменными | Колледж Алгебра |
Вычисление определителя матрицы 3 × 3
Найти определитель матрицы 2×2 несложно, но найти определитель матрицы 3×3 сложнее. Один из методов состоит в том, чтобы дополнить матрицу 3×3 повторением первых двух столбцов, получив матрицу 3×5. Затем мы вычисляем сумму произведений записей на по каждой из трех диагоналей (слева вверху справа внизу) и вычитаем произведения записей до по каждой из трех диагоналей (слева снизу вверх справа). Это легче понять с визуальным и пример.
От верхнего левого угла к нижнему правому: умножьте числа по первой диагонали. Прибавьте результат к произведению записей по второй диагонали. Добавьте этот результат к произведению записей вниз по третьей диагонали.
Из нижнего левого угла в верхний правый: вычтите произведение записей вверх по первой диагонали. Из этого результата вычтите произведение вхождений вверх по второй диагонали. Из этого результата вычтите произведение вхождений вверх по третьей диагонали.
Рисунок 2
Алгебра выглядит следующим образом: c}_{3}+{b}_{1}{c}_{2}{a}_{3}+{c}_{1}{a}_{2}{b}_{3} -{a}_{3}{b}_{2}{c}_{1}-{b}_{3}{c}_{2}{a}_{1}-{c}_{ 3}{a}_{2}{b}_{1}∣A∣=a1b2c3+b1c2a3+c1a2b3−a3b2c1−b3 c2a1−c3a2b1
Можно ли использовать тот же метод для нахождения определителя большей матрицы?
Нет, этот метод работает только для
2 × 22\text{ }\times \text{ }22 × 2
и
3 × 3\text{3}\text{ }\times \text{ }33 × 3
матрицы. Для больших матриц лучше всего использовать графическую утилиту или компьютерное программное обеспечение.
Использование правила Крамера для решения системы трех уравнений с тремя переменными
Теперь, когда мы можем найти определитель матрицы 3 × 3, мы можем применить правило Крамера для решения системы трех уравнений с тремя переменными . Правило Крамера является простым и следует шаблону, согласующемуся с правилом Крамера для матриц 2 × 2. Однако по мере увеличения порядка матрицы до 3 × 3 требуется гораздо больше вычислений.
Когда мы вычисляем, что определитель равен нулю, правило Крамера не указывает, имеет ли система решение или бесконечное число решений. Чтобы выяснить это, мы должны выполнить исключение в системе.
Мы знаем, что определитель, равный нулю, означает, что либо система не имеет решений, либо имеет бесконечное число решений. Чтобы увидеть, какой из них, мы используем процесс исключения. Наша цель — исключить одну из переменных.
Сборник задач с решениями и ответами — Микроэкономика — Балакина Т.П.: Экономика — PDF (77690)
PDF-файл из архива «Сборник задач с решениями и ответами — Микроэкономика — Балакина Т.П.»,
который расположен в категории «».
Всё это находится в предмете «экономика» из 3 семестр, которые можно найти в файловом архиве МФТИ (ГУ).
Не смотря на прямую связь этого архива с МФТИ (ГУ), его также можно найти и в других разделах. .
ВЫСШАЯ ШКОЛА ЭКОНОМИКИНАЦИОНАЛЬНЫЙ ИССЛЕДОВАТЕЛЬСКИЙ УНИВЕРСИТЕТТ.П.Балакина, Е.А.Левина, Е.В.Покатович, Е.В.ПоповаМИКРОЭКОНОМИКАпромежуточныйуровеньСборник задачс решениями и ответамиРекомендовано УМО в области экономики,менеджмента, логистики и бизнес-информатикив качестве учебного пособия для студентов,обучающихся по направлениям «Экономика»,«Менеджмент», «Бизнес-информатика»и специальности «Логистика»Издательский дом Высшей школы экономикиМосква 2013УДК 330.101.542ББК 65.012.1М59Р е ц е н з е н т ы:доктор экономических наук, профессор кафедры«Инвестиции и инновации» Финансового университетапри Правительстве Российской Федерации Д. И.
Кокурин;доктор экономических наук, зав. кафедройэкономической теории НИУ ВШЭ, профессор С.Ф. Серегина;доктор экономических наук, зам. зав. кафедройэкономической теории НИУ ВШЭ Т.В. КулаковаМикроэкономика: промежуточный уровень. Сборник задачМ59 с решениями и ответами [Текст] : учеб. пособие / Т. П. Балакина,Е. А. Левина, Е. В. Покатович, Е. В. Попова ; Нац. исслед.
ун-т«Высшая школа экономики». — М. : Изд. дом Высшей школы экономики, 2013. — 503, [1] с. — 1000 экз. — ISBN 978-5-7598-0983-8(в обл.).Учебное пособие содержит подборку заданий по основным разделаммикроэкономики (выбор потребителя в условиях определенности и вусловиях неопределенности, поведение производителя, общее равновесие, провалы рынка, рыночные структуры), помогающих читателям научиться применять полученные базовые знания по предмету.
. . . . . . .227237246306323323330334341343385390390394397398404407407409409410411416444Г л а в а 6. Выбор потребителя в условиях неопределенности 4486.1. Денежные лотереи, отношение к риску, денежный эквивалент лотереи и премия за риск, функция ожидаемой полезности . . . . . . . . . . . . . . . . .
. . . . . . . . . . . . . . . . . . . . . . . . . . . . . . . . . . . . . . 503Используемая литература . . . . . . . . . . . . . . . . . . . . . . . . . . . . . . 503ПредисловиеКафедра микроэкономического анализа Национального исследовательского университета «Высшая школа экономики» существует уже более десяти лет, однако до сих пор не был издансборник задач, соответствующий бакалаврским программам кафедры.
Понятно, что такое положение вещей огорчало студентов,которые хотели бы иметь возможность решать как можно большее количество задач, так как именно умение решать задачирассматривается в качестве критерия освоения курса. У каждого лектора — свой стиль подачи материала и свои методикипреподавания, которые, к тому же, меняются в зависимости отаудитории слушателей. И все же есть базовый материал, которыйдолжен быть освоен при любом подходе к изложению курсасовременной микроэкономики.
продвинутый уровень видео-лекций от Олимп
Продвинутая Микроэкономика
Вы углубите свои знания по микроэкономике и научитесь решать сложные задачи.
Подойдет тем, кто занимается экономикой от полугода, планирует в этом сезоне добиться высоких результатов на перечневых олимпиадах и пройти на заключительный этап Всероссийской олимпиады по экономике.
Начать учиться
3 900₽
Из чего состоит курс
8 недель подготовки
16 Видео-лекций с разбором теории от ведущих преподавателей из МГУ и ВШЭ
Тесты для тренировки к отборочным этапам олимпиад
Задачи для самостоятельного решения по каждой теме
Старший преподаватель Высшей Школы Экономики. Лауреат премии «Золотая Вышка»
Победитель конкурса «Учитель года Москвы»
Главный преподаватель многих выездных школ
Александр Челеховский
Дмитрий Блидман
Антон Случ
Преподаватели курса
Дмитрий Очков
Шестикратный призёр ВОШ по экономике и математике
Составитель задач Открытого Чемпионата школ по экономике
Преподаватель лицея НИУ ВШЭ, Основатель Школы Профессий N1
Преподаватель экономики в 57 школе
Тимур Магжанов
Михаил Гойхман
Анна Самоделкина
Татьяна Рафикова
Преподаватель ЭМШ МГУ 2014-2015
Уже 3 года Олимп занимается подготовкой школьников к олимпиадам. В нашей команде работают учителя лучших московских школ и вузов, организаторы выездных школ, победители Всероссийской олимпиады школьников и студенты лучших программ.
Эффективность наших курсов подтверждают результаты учеников — всего в копилке Олимпа уже более 800 дипломов.
Процесс обучения
Рассказываем, как вы будете учиться на каждой из недель курса
Получаете еженедельные видео-лекции с теорией и семинары с разборами решения задач.
Самостоятельно решаете тесты после лекций и большое домашнее задание в конце темы.
Смотрите разборы наиболее сложных задач.
Общаетесь с другими учениками, отдыхаете и играете в экономические игры.
Темы курса
Нажмите на +, чтобы узнать подробнее
Стоимость
Продвинутая Микроэкономика
• 8 семинаров с разбором методов решения задач
• Тестовые задания для проработки навыков
• Задачи для самостоятельного решения по каждой теме
• 16 Видео-лекций с разбором теории от ведущих преподавателей МГУ и ВШЭ
3 900₽
Записаться
Нажмите на +, чтобы узнать подробнее
Георгий Кузьмин
Своё изучение олимпиадой экономики я начал с Олимпа летом после окончания десятого класса. За год я прошёл все курсы: базовый, продвинтуый и профессиональный. Вся необходимая теория для решения задач рассказывается в лекциях очень понятно и лаконично. А задачи в подборках подобраны настолько оптимально, что прорешав их все, можно будет решить практически любую задачу на олимпиаде. Олимп позволяет за год изучить олимпиадную экономику с нуля и выиграть заветный диплом всош! Всем рекомендую этот проект!
https://vk.com/gregory_kuzmin
Когда я только услышал об Олимпе и преподавательском составе, я сразу понял, что это будет очень продуктивно, потому что большинство людей я знал как сильнейших преподавателей. Так оно и оказалось. Сочетание онлайн-лекций с возможностью пересматривать непонятные моменты по несколько раз и отличные подборки новых задач позволили подготовиться ко всеросу. На мой взгляд, это уникальный проект, которому нет пока что равных в России, поскольку такую доступность занятий для регионов (в том числе по соотношению цена-качество) не предоставляет ни одна школа.
https://vk.com/id10041306
Егор Четвериков
Изначально меня не интересовали лекции, скорее я желала порешать новые и интересные задачи. Но в итоге я получала пользу и от одного, и от другого. Мне понравилось, что лекции рассчитаны на разную аудиторию. В некоторых темах я совсем не разбиралась и было приятно, что объясняют с нуля и очень понятно. Но при этом были и достаточно сложные лекции. Задачи, как я и ожидала, были новые и интересные. Их решение и оформление очень помогло мне на заключительном этапе ВОШ. Я стала призером и в этом большая заслуга Олимпа, так что я очень благодарна.
https://vk.com/v_juliana_v
Юлиана Вотякова
Отзывы ❤️
Смотреть больше отзывов
Акция
Сэкономьте до 30% при покупке сразу нескольких курсов!
8 500 ₽
(6 месяцев обучения)
Подробнее
Пакет «Полный»
Подробнее
6 500₽
(4 месяца обучения)
Пакет «Продвинутый»
Click to order
Микроэкономика
Справка по микроэкономике почти обязательна для большинства студентов. Университеты требуют очень многого, и курсы, как правило, чрезвычайно интенсивны. В соответствии с этим профессиональная онлайн-помощь по микроэкономике может помочь вам сэкономить время и лучше уделить внимание более полезным для вас вещам. Не все, но некоторые задания и задачи требуют теоретической подготовки, даже если она не представляет большого интереса. Чтобы избежать этого большого количества чтения теории, вы можете принять правильное решение и довериться нашим микроэкономическим услугам и высококвалифицированным экспертам.
Люди ежедневно сталкиваются с применимыми понятиями в этой области, потому что микроэкономика очень динамичная наука. Таким образом, знания, необходимые для хорошего понимания микроэкономики, можно построить, используя глубокий и внимательный взгляд на текущие жизненные ситуации и на реальных примерах. Онлайн-задания по микроэкономике — очень полезный способ сделать это очень быстро. Мы предоставим вам помощь по микроэкономике той степени и темы, которая вам действительно нужна, и вы получите свою работу в срок без дополнительных усилий. Все, что вам нужно, это отправить свой запрос и тему. Мы сделаем все остальное!
Почему стоит выбрать онлайн-помощь по микроэкономике?
Микроэкономика — это комплексное исследование, которое требует большого количества навыков, и мы можем обеспечить это на приемлемых условиях!
Квалифицированные и опытные специалисты в области микроэкономики;
Новейшая информация;
Безопасные способы оплаты;
Доступные цены, отвечающие высочайшему качеству и срокам;
Скидки постоянным клиентам;
Проекты по микроэкономике выполняются очень быстро с помощью наших онлайн-сервисов по микроэкономике;
У нас есть решение любой микроэкономической проблемы. Как показали недавние финансовые кризисы, попытка понять, как работает микроэкономика, может сбить с толку даже самые яркие умы. Все наши специалисты имеют многолетний и ценный опыт работы над сложными идеями и доведения их до клиентов.
Ваше домашнее задание по микроэкономике очень важно, и наша миссия состоит в том, чтобы извлечь из него максимум пользы и ответить на все микроэкономические вопросы, которые возникают у вас в голове. Не стесняйтесь спрашивать и консультироваться с нашими экспертами по домашней работе по микроэкономике.
Если вы столкнулись с трудностями при выполнении заданий по микроэкономике, то вы обратились по адресу!
быстрые решения – мы готовы справиться с любой вашей задачей!
мы предоставляем онлайн-помощь для всех по доступной цене,
вы всегда можете получить консультацию и помощь наших специалистов по микроэкономике онлайн!
Так что, если домашнее задание кажется вам слишком сложным, а срок слишком близок, но вам нужно, чтобы оно было готово вовремя, это определенно означает, что вам нужна срочная и профессиональная онлайн-помощь по микроэкономике!
Наши специалисты по микроэкономике проверят вашу домашнюю работу и вернут ее как можно скорее по лучшей цене. Мы обеспечиваем лучшее качество работы и 100% гарантию удовлетворения.
Мы верим в вас, так что продолжайте верить и в нас!
Решения центральных проблем экономики – Микроэкономика Класс 11
Решения центральных проблем экономики
Основные проблемы экономики – (1) что производить? (2) как производить? и (3) для кого производить? являются общими проблемами в различных экономиках, независимо от их размера и местоположения. Решения центральных проблем экономики могут быть достигнуты путем принятия определенных механизмов в различных типах экономики.
Решения центральных проблем экономики – капиталистическая экономика
Капиталистическая экономика дает решение центральных проблем экономики через ценовой механизм,
Решения центральных проблем экономики – капиталистическая экономика
Социалистическая экономика дает решение центральных проблем экономики через планирование, а в смешанной экономике через ценовой механизм и экономическое планирование.
Рыночная или капиталистическая экономика
Рыночная экономика – это свободная экономика. Всякая деятельность в рыночной экономике находится под влиянием мотива получения прибыли. Производители свободны в выборе решения центральных проблем экономики: «что производить?», «как производить?» и «для кого производить?». Производители принимают это решение на основе спроса и предложения на рынке исключительно с целью максимизации прибыли.
Производитель будет производить те товары, на которые есть большой спрос, но мало предложений. Такие товары принесут производителю более высокую цену и прибыль. Цена товара определяется взаимодействием сил спроса и предложения, т. е. между потребителями и производителями на товарном рынке. Производитель будет использовать те ресурсы, которые будут поддерживать издержки производства на возможно более низком уровне, чтобы он мог получать более высокую прибыль. Также производители будут производить товары для тех людей, которые могут позволить себе платить высокую цену. Более бедные слои общества часто игнорируются.
Рабочие выбирают работу с максимальной оплатой труда. Потребитель также предпочитает покупать те товары, которые обеспечивают максимальное удовлетворение. Точно так же, как цена товара, цена факторов производства определяется на рынке факторов производства.
Социалистическая экономика
В централизованно планируемой или социалистической экономике все решения, касающиеся основных проблем экономики, принимаются назначенным правительством или какой-либо центральной властью. Все решения принимаются с целью максимизации общественного благосостояния. Центральная власть принимает плановую экономику развития как решение центральных проблем экономики. Производятся те товары и услуги, которые, по мнению центральной власти, будут в наилучших интересах общества.
Будет принята более социально полезная техника производства. Например, в ситуации массовой безработицы будут применяться трудоемкие технологии, а не капиталоемкие. Доступные товары будут производиться для более бедных слоев общества, даже если они не приносят прибыли. Социальное благополучие является более высоким приоритетом, чем максимизация прибыли.
Подход смешанной экономики к решению основных проблем экономики
Смешанная экономика сочетает в себе достоинства как капиталистической, так и социалистической экономики. Социальное благополучие, а также рыночные силы учитываются при принятии решений, касающихся решения центральных проблем экономики. В некоторых областях производства производители свободны в принятии решений в соответствии с рыночными силами с целью максимизации прибыли.
Векторы для чайников. Часть 1. Сложение, разность, умножение на число. — Блог
Векторы для чайников. Часть 1. Сложение, разность, умножение на число.
18 июня 2021
0
Marina Pashnina
Есть на баше одна уже старая шутка про черного кота, которая звучит как-то так:
— Если чёрный кот перешел дорогу туда и обратно, это значит, что он удвоил наказание или отменил своё решение? — Кот скалярный или векторный? Если скалярный — то удвоил, если векторный — то отменил.
В общем, с этой ноты и начинается статья про то, что было бы, если бы уже знакомый нам кот по имени Котаненс был векторным или скалярным, или статья о векторах.
Вектор — это направленный отрезок и главное, что нужно знать о векторе — у него есть величина и направление. Тут пока все сходится с котом, переходящим дорогу: кот идет в определенном направлении и проходит при этом определенное расстояние.
В школьном курсе геометрии рассматриваются некоторые действия над векторами: сложение векторов, умножение вектора на число, разность векторов. В основном, эти действия интуитивно понятные, достаточно только представить или нарисовать вектор или пару векторов. Давайте коротко рассмотрим эти действия.
Отдельно оговорюсь о существовании нулевых векторов — таких векторов, у которых начало и конец находятся в одной точке. Для упрощения материала этот нулевой вектор будет местами игнорироваться ввиду малой практической значимости.
Что ж, начнем.
Сложение векторов
Есть несколько методов сложения векторов, которые руководствуются похожими принципами.
Для сложения двух векторов нам понадобятся вектора ā и b̅ (кто бы мог подумать?) .
Отложим вектор b̅ от вектора ā и проведем от конца вектора ā до начала вектора b̅ результирующий вектор. Этот прием называется правило треугольника.
Результатом сложения будет вектор ā + b̅. Всё также, как с котом: сначала кот прошел по вектору ā (определенное расстояние в определенном направлении), затем по вектору b̅. То, что он прошел в итоге — это и есть результирующий вектор ā + b̅.
Это же работает и для сложения нескольких векторов: кот может пробежать по зиг-загу, или же статно пройти по результирующему вектору.
Если же вектор b̅ отложить не от конца, а от начала вектора ā, то получится правило параллелограмма.
Тут можно вспомнить векторного кота из шутки — если векторный кот пройдет туда-обратно, результирующий вектор, по которому он пройдет — будет равняться 0, а значит — кот отменит свое проклятье.
Скалярный же кот при проходе туда-обратно сложит длины векторов и получит число в 2 раза больше изначального, а значит и проклятье удвоится.
Разность векторов
Разность векторов ā и b̅ также можно рассчитать несколькими способами.
ā — b̅, как частный случай сложения — это сложение вектора ā с вектором, обратному b̅, т.е. ā + (-b̅).
Вектор -b̅, обратный к вектору b̅ сделать просто: кот просто должен пойти в обратную сторону.
А дальше просто складываем этот вектор с вектором ā.
Второй способ получить разность векторов чуть сложнее для осознания: разностью векторов ā и b̅ называется такой вектор, сумма которого с вектором b̅ дает вектор ā. Для понимания достаточно просто нарисовать на листочке и все станет ясно.
Умножение вектора на число
Умножение вектора a на число n создает такой вектор, длина которого равна |ā| * | n |, где |ā| — это длина вектора a, а направление сохраняется при n >= 0 и меняется при n < 0.
Эта статья оказалась достаточно объемной, поэтому я рещила разделить ее на 2 части: во второй части статьи будет рассказано про векторное и скалярное произведение векторов и об этом можно почитать в статье «Векторы для чайников. Часть 2.».
Операции с векторами, сложение векторов, умножение вектора на действительное число.
Рассмотрим вектор v с начальной точкой в начале координат в любой координатной системе x-y и с конечной точкой в (a,b). Мы говорим, что вектор находится в стандартном положении и ссылаемся на него как на радиус-вектор. Обратите внимание, что пара точек определяет этот вектор. Таким образом, мы можем использовать это для обозначения вектора. Чтобы подчеркнуть, что мы имеем в виду вектор, и, чтобы избежать путаницы, как правило, пишут: v = .
Координата a есть скаляром горизонтальной компоненты вектора, и координата b есть скаляром вертикальной компоненты вектора. Под скаляром мы подразумеваем численное количество, а не векторную величину. Таким образом, это рассматривается как компонентная форма v. Обратите внимание, что a и b НЕ вектора и их не надо путать с определением компонента вектора.
Теперь рассмотрим с A = (x1, y1) и C = (x2, y2). Давайте рассмотрим, как найти радиус вектор, эквивалентный . Как Вы видите на рисунке внизу, начальная точка A перемещена в начало координат (0, 0). Координаты P находятся вычитанием координат A из координат C. Таким образом, P = (x2 — x1, y2 — y1) и радиус вектор есть .
Можно показать, что и имеют одну и ту же величину и направление, и поэтому эквивалентны. Таким образом, = = 2 — x1, y2 — y1 >.
Компонентная форма с A = (x1, y1) и C = (x2, y2) есть = 2 — x1, y2 — y1 >.
Пример 1 Найдите компонентную форму если C = (- 4, — 3) и F = (1, 5).
Решение Мы имеем = = .
Обратите внимание, что вектор есть равным радиус-вектору , как показано на рисунке вверху.
Теперь, когда мы знаем, как записать вектор в компонентной форме, давайте изложим некоторые определения. Длину вектора v легко определить, когда известны компоненты вектора. Для v = 1, v2 >, мы имеем |v|2 = v21 + v22Используя теорему Пифагора |v| = √v21 + v22.
Длина, или величина ветктора v = 1, v2 > находится как |v| = √v21 + v22.
Два вектора равны или эквивалентны, если они имеют одну и ту же величину и одно и то же направление.
Пусть u = 1, u2 > и v = 1, v2 >. Tогда 1, u2 > = 1, v2 > только если u1 = v1 and u2 = v2.
Операции с векторами
Чтобы умножить вектор V на положительное число, мы умножаем его длину на это число. Его направление остается прежним. Когда вектор V умножается на 2, например, его длина увеличивается в два раза, но его направление не изменяется. Когда вектор умножается на 1,6, его длина увеличивается на 60%, а направление остается прежним. Чтобы умножить вектор V на отрицательное действительное число, умножаем его длину на это число и изменяем направление на противоположное. Например, Когда вектор умножается на (-2), его длина увеличивается в два раза и его направление изменяется на противоположное.
Так как действительные числа работают как скалярные множители в умножении векторов, мы называем их скаляры и произведение kv называется скалярные кратные v.
Для действительного числа k и вектора v = 1, v2 >, скалярное произведение k и v есть kv = k.1, v2 > = 1, kv2 >. Вектор kv есть скалярным кратным вектора v.
Пример 2 Пусть u = и w = . Найдите — 7w, 3u и — 1w.
Теперь мы можем сложить два вектора, используя компоненты. Чтобы сложить два вектора в компонентной форме, мы складываем соответствующие компоненты. Пусть u = 1, u2 > и v = 1, v2 >. Тогда u + v = 1 + v1, u2 + v2 >
Например, если v = и w = , тогда v + w = =
Если u = 1, u2 > и v = 1, v2 >, тогда u + v = 1 + v1, u2 + v2 >.
Перед тем, как мы определим вычитание векторов нам нужно дать определение — v. Противоположный вектору v = 1, v2 >, изображенному внизу, есть — v = (- 1).v = (- 1)1, v2 > = 1, — v2 >
Вычитание векторов, такое как u — v вовлекает вычитание соответствующих компонент. Мы покажем это представлением u — v как u + (- v). Если u = 1, u2 > и v = 1, v2 >, тогда u — v = u + (- v) = 1, u2 > + 1, — v2 > = 1 + (- v1), u2 + (- v2) > = 1 — v1, u2 — v2 >
Мы можем проиллюстрировать вычитание векторов с помощью параллелограмма , как мы это делали для сложения векторов.
Вычитание векторов
Если u = 1, u2 > и v = 1, v2 >, тогда u — v = 1 — v1, u2 — v2 >.
Интересно сравнить суммы двух векторов с разницей тех же двух векторов в одном параллелограмме. Векторы u + v и u — v есть диагоналями параллелограмма.
Пример 3 Сделайте следующие вычисления, где u = и v = . a) u + v b) u — 6v c)3u + 4v d)|5v — 2u|
Прежде чем сформулировать свойства векторного сложения и умножения, мы должны дать определение еще одному специальному вектору — нулевому вектору. Вектор, чья начальная точка совпадает с конечной точкой, называется нулевым вектором, обозначается O, или . Его величина равна 0. В сложении векторов: v + O = v. 1, v2 > + = 1, v2 > Операции над векторами обладают те же самыми свойствами, что и операции над вещественными числами.
Свойства векторного сложения и умножения
Для всех векторов u, v, и w, и для всех скаляров b и c: 1. u + v = v + u. 2. u + (v + w) = (u + v) + w. 3. v + O = v. 4 1.v = v; 0.v = O. 5. v + (- v) = O. 6. b(cv) = (bc)v. 7. (b + c)v = bv + cv. 8. b(u + v) = bu + bv.
Орты
Вектор величиной, или длиной 1 называется орт. Вектор v = есть орт, потому что |v| = || = √(- 3/5)2 + (4/5)2 = √9/25 + 16/25 = √25/25 = √1 = 1.
Пример 4 Найдите орт, который имеет то же самое направление, что и вектор w = .
Решение Найдем сначала длину w: |w| = √(- 3)2 + 52 = √34. Таким образом, мы ищем вектор, с длиной 1/√34 от w и с таким же самым направлением, что и вектор w. Этот вектор есть u = w/√34 = /√34 = . Вектор u есть орт, потому что |u| = |w/√34| = = √9/34 + 25/34 = √34/34 = √1 = 1.
Если v есть вектор и v ≠ O, тогда (1/|v|)• v, or v/|v|, есть орт в направлении v.
Хотя орты могут иметь любое направление, орты, параллельные осям x и y особенно полезны. Они определяются как i = and j = .
Любой вектор может быть выражен как линейная комбинация орта i и j. Например, пусть v = 1, v2 >. Tогда v = 1, v2 > = 1, 0 > + 2 > = v1 + v2 = v1i + v2j.
Пример 5 Выразите вектор r = как линейную комбинацию i и j.
Решение r = = 2i + (- 6)j = 2i — 6j.
Пример 6 Запишите вектор q = — i + 7j в компонентной форме.
Решениеq = — i + 7j = -1i + 7j =
Векторные операции могут быть также выполнены, когда векторы записаны как линейные i и j.
Пример 7 Если a = 5i — 2j и b = -i + 8j, найдите 3a — b.
Решение 3a — b = 3(5i — 2j) — (- i + 8j) = 15i — 6j + i — 8j = 16i — 14j.
Углы обзора
Конечная точка P орты в стандартной позиции есть точкой на единичной окружности, определенной (cosθ, sinθ). Таким образом, орт может быть выражен в компонентной форме, u = , или как линейная комбинация орт i и j, u = (cosθ)i + (sinθ)j, где компоненты u есть функциями угла обзора θ измеряемого против часовой стрелки от оси x к этому вектору. Так как θ изменяется от 0 до 2π, точка P отслеживает круг x2 + y2 = 1. Это охватывает все возможные направления ортов и тогда уравнение u = (cosθ)i + (sinθ)j описывает каждый возможный орт на плоскости.
Пример 8 Вычислите и сделайте эскиз орта u = (cosθ)i + (sinθ)j для θ = 2π/3. Изобразите единичную окружность на эскизе.
Решение u = (cos(2π/3))i + (sin(2π/3))j = (- 1/2)i + (√3/2)j
Пусть v = 1, v2 > с углом обзора θ. Используя определение функции тангенса, мы можем определить угол обзора их компонент v:
Пример 9 Определите угол обзора θ вектора w = — 4i — 3j.
Решение Мы знаем, что w = — 4i — 3j = . Таким образом, имеем tanθ = (- 3)/(- 4) = 3/4 и θ = tan— 1(3/4). Так как w находится в третьем квадранте, мы знаем, что θ есть углом третьего квадранта. Соответствующий угол есть tan— 1(3/4) ≈ 37°, и θ ≈ 180° + 37°, или 217°.
Это удобно для работы с прикладными задачами, а в последующих курсах, чтобы иметь способ выразить вектор так, чтобы его величина и направление могли быть легко определены или прочитаны. Пусть v это вектор. Тогда v/|v| есть орт в том же самом направлении, что и v. Таким образом, мы имеем v/|v| = (cosθ)i + (sinθ)j v = |v|[(cosθ)i + (sinθ)j] Умножая на |v| v = |v|(cosθ)i + |v|(sinθ)j.
Углы между векторами
Когда вектор умножается на скаляр, результатом есть вектор. Когда складываются два вектора, результатом также есть вектор. Таким образом, мы могли бы ожидать, что произведение двух векторов есть вектор, но это не так. Скалярное произведение двух векторов есть действительное число или скаляр. Этот результат полезен в нахождении угла между двумя векторами и в определении, являются ли два вектора перпендикулярными.
Скалярное произведение двух векторов u = 1, u2 > и v = 1, v2 > is u • v = u1.v1 + u2.v2 (Обратите внимание, что u1v1 + u2v2 есть скаляром, а не вектором.)
Пример 10Найдите скалярное произведение, когда u = , v = и w = . a)u • w b)w • v
Решение a) u • w = 2(- 3) + (- 5)1 = — 6 — 5 = — 11; b) w • v = (- 3)0 + 1(4) = 0 + 4 = 4.
Скалярное произведение может быть использовано для нахождения угла между двумя векторами. Угол между двумя векторами это самый маленький положительный угол, образованный двумя направленными отрезками. Таким образом, θ между u и v это тот же самый угол, что и между v и u, и 0 ≤ θ ≤ π.
Если θ есть углом между двумя ненулевыми векторами u и v, тогда cosθ = (u • v)/|u||v|.
Пример 11Найдите угол между u = и v = .
Решение Начнем с нахождения u • v, |u|, и |v|: u • v = 3(- 4) + 7(2) = 2, |u| = √32 + 72 = √58, and |v| = √(- 4)2 + 22 = √20. Tогда cosα = (u • v)/|u||v| = 2/√58.√20 α = cos— 1(2/√58.√20) α ≈ 86,6°.
Равновесие сил
Когда несколько сил действуют на одну и ту же точку на объекте, их векторная сумма должна быть равна нуля, для того, чтобы был баланс. Когда есть баланс сил, то объект является стационарным или движется по прямой линии, без ускорения. Тот факт, что векторная сумма должна быть равна нулю вывода для получения баланса, и наоборот, позволяет решать нам многие прикладные задачи с участием сил.
Пример 12 Подвесной блок 350- фунтовый блок подвешен с помощью двух кабелей. осталось. В точке А есть три силы, действующие так: W блок тянет вниз, а R и S (два кабеля) тянут вверх и наружу. Найдите нагрузку каждого кабеля.
Решение Нарисуем диаграмму с начальными точками каждого вектора в начале кооординат. Для баланса, сумма векторов должна быть равна О:
R + S + W = О. Мы можем выразить каждый вектор через его величину и угол обзора : R = |R|[(cos125°)i + (sin125°)j], S = |S|[(cos37°)i + (sin37°)j], и W = |W|[(cos270°)i + (sin270°)j] = 350(cos270°)i + 350(sin270°)j = -350j cos270° = 0; sin270° = — 1. Заменяя R, S, и W in R + S + W + O, мы имеем [|R|(cos125°) + |S|(cos37°)]i + [|R|(sin125°) + |S|(sin37°) — 350]j = 0i + 0j. Это дает нам систему уравнений: |R|(cos125°) + |S|(cos37°) = 0, |R|(sin125°) + |S|(sin37°) — 350 = 0. Решая эту систему, мы получаем |R| ≈ 280 и |S| ≈ 201. Таким образом, нагрузка на кабели 280 фунтов и 201 фунт.
Что такое результат?
Результирующее число представляет собой векторную сумму двух или более векторов. Это результат сложения двух или более векторов вместе. Если сложить векторы смещения A, B и C, результатом будет вектор R. Как показано на диаграмме, вектор R можно определить с помощью точно нарисованной масштабированной диаграммы сложения векторов.
Сказать, что вектор R есть результирующее перемещение векторов смещения A, B и C означает, что человек, который шел со смещением A, затем B, а затем C, сместится на ту же величину, что и человек, который шел со смещением R. Вектор смещения R дает то же самое результат в виде векторов смещения A + B + C. Поэтому можно сказать, что
A + B + C = R
Вышеприведенное обсуждение относится к результату сложения векторов смещения. При добавлении векторов смещения результатом будет результирующее смещение . Но любые два вектора могут быть добавлены, если они являются одной и той же векторной величиной. Если добавить два или более векторов скорости, результатом будет результирующая скорость . Если добавить два или более векторов силы, то результатом будет равнодействующая сила . Если сложить два или более векторов импульса, то результатом будет …
Во всех таких случаях результирующий вектор (будь то вектор смещения, вектор силы, вектор скорости и т. д.) является результатом сложения отдельных векторов. Это то же самое, что добавить А + В + С + … . «Делать A + B + C — это то же самое, что делать R». В качестве примера рассмотрим футболиста, которого одновременно бьют три игрока противоположной команды (игроки A, B и C). Футболист испытывает на себе воздействие трех различных сил. Каждая приложенная сила вносит свой вклад в общую или результирующую силу. Если сложить три силы вместе с помощью методов сложения векторов (обсуждавшихся ранее), то можно определить результирующий вектор R. В этом случае испытать три силы A, B и C — это то же самое, что испытать силу R. Если вас ударят игроки A, B и C, это приведет к той же силе, что и удар, нанесенный одним игроком, применяющим силу R». Сделать A + B + C — это то же самое, что сделать R». Вектор R — это тот же результат, что и векторы A + B + C!!
Таким образом, результат представляет собой векторную сумму всех отдельных векторов. Результат является результатом объединения отдельных векторов вместе. Результирующую можно определить путем сложения отдельных сил с использованием методов сложения векторов.
Мы хотели бы предложить …
Иногда недостаточно просто прочитать об этом. Вы должны взаимодействовать с ним! И это именно то, что вы делаете, когда используете один из интерактивных материалов The Physics Classroom. Мы хотели бы предложить вам совместить чтение этой страницы с использованием нашей интерактивной игры Name That Vector, нашей интерактивной игры с добавлением вектора или нашей интерактивной игры с угадыванием вектора. Все три интерактива можно найти в разделе Physics Interactive на нашем веб-сайте, и они обеспечивают интерактивный опыт с возможностью добавления векторов.
Посетите: Назовите этот вектор | Добавление вектора | Игра «Угадай вектор»
Следующий раздел:
Перейти к следующему уроку:
Добавление вектора
Математика и наука были изобретены людьми для описания и
понимать окружающий мир.
Заметим, что существуют некоторые величины и процессы в
наш мир, который зависит от направление в котором
они происходят, и есть некоторые величины, которые не зависят
по направлению.
Математики и ученые называют количество
который зависит от направления векторной величины . Количество
которая не зависит от направления, называется скалярной величиной . А
векторное количество
имеет две характеристики: величина и направление . Когда
сравнение
две векторные величины одного и того же типа, вы должны сравнить обе
величина и направление.
На этом слайде мы описываем метод сложения двух векторов.
Сложение векторов — это один из аспектов большой векторной алгебры, которую мы изучаем. , а не будут представлены на этом сайте. Добавление вектора представлено
здесь, потому что это встречается довольно часто при изучении движения
и потому что он демонстрирует некоторые фундаментальные различия между
векторы и скаляры.
Векторы обычно обозначаются на рисунках стрелкой.
Длина стрелки указывает величину и
кончик стрелки указывает направление. Вектор
помечены буквой в алфавитном порядке
буква с линией сверху, чтобы отличить ее от скаляра.
Величину вектора будем обозначать символом |а| . Направление
будет измеряться углом фи относительно координаты
ось х . Ось координат y перпендикулярна х . Примечание: Оси координат x и y сами по себе
векторы! Они имеют величину и направление. Сначала ты
столкнуться с осями координат, когда вы учитесь строить графики. Так что у тебя есть
использовал векторы в течение некоторого времени, даже не подозревая об этом!
Если мы построим пунктирную линию от кончика вектор а идущий параллельно оси х, он пересекает ось у в том месте, где мы
этикетка или . Точно так же линия от кончика вектора
параллельно оси y пересекает ось x в точке по оси .
Величины x и ay называются
компоненты
вектора, и оба являются скалярными квантитами.
Чтобы добавить два вектора, a и b ,
мы сначала разбиваем каждый вектор на его компоненты, x и a ,
и бх и по ,
как показано на рисунке. Из правил, регулирующих
равенство
векторов, синий вектор b равен черному вектору b потому что он имеет одинаковую равную длину и одинаковое направление. Теперь, поскольку компоненты вектора и вектор b являются скалярами, мы можем добавить x-компоненты для генерации
x-компонент нового вектора c :
сх = топор + Ьх
Точно так же мы можем добавить y-компонентов :
су = ау + по
Новые компоненты cx и cy полностью определяют
новый вектор c , указав как величину, так и направление.
Внимательно взглянув на диаграмму, мы видим, что сложение двух векторов дает
новый вектор, который равен , а не в направлении любого из
исходные векторы, величина которых равна , а не и равна сумме
величин исходных векторов.
Векторная алгебра сильно отличается от скалярной алгебры, потому что она должна
учитывать как величину, так и направление.
Примечание: На этом слайде для простоты мы разработали
компоненты только в двух измерениях; имеются две оси координат.
В действительности существуют три пространственных измерения и три компонента мира.
Решите систему уравнений x в квадрате минус 5 игрек в минус 24 равняется нулю игрек равняется икс минус 2 — вопрос №2742977
Ответов пока нет
Михаил Александров
от 0 p.
Читать ответы
science
от 300 p.
Читать ответы
Андрей Андреевич
от 70 p.
Читать ответы
Посмотреть всех экспертов из раздела Учеба и наука
Пользуйтесь нашим приложением
2-5y-14=0 Tiger Algebra Solver
Пошаговое решение :
Шаг 1 :
Попытка факторизовать путем разделения среднего члена , y
2 его коэффициент равен 1 . Средний член равен -5y, его коэффициент равен -5. Последний член, «константа», равен -14
Шаг-1: Умножьте коэффициент первого члена на константу 1 • -14 = -14 равен коэффициенту среднего члена, который равен -5 .
-14
+
1
=
-13
-7
+
2
=
-5
That’s it
Шаг 3 : Перепишите полином, разделяющий средний член, используя два множителя, найденные на шаге 2 выше, -7 и 2 0011
Шаг 4 : Сложите первые 2 члена, выделив одинаковые множители : 5 : Сложите четыре условия шага 4 : (y+2) • (y-7) Какая нужна факторизация
Уравнение в конце шага 1 • — 1 :
4 (y + 7) = 0
Этап 2 :
Теория – корни продукта :
2.1 Произведение нескольких слагаемых равно нулю.
Если произведение двух или более слагаемых равно нулю, то хотя бы одно из слагаемых должно быть равно нулю.
Теперь мы будем решать каждый термин = 0 отдельно
Другими словами, мы собираемся решить столько уравнений, сколько членов в произведении
Любое решение термина = 0 также решает произведение = 0.
Решение уравнения с одной переменной :
2.2 Решение : y+2 = 0
Вычитание 2 с обеих сторон уравнения: y = -2
Решение единого переменного уравнения:
2,3 Решение: Y -7 = 0
Добавить 7 к обеим сторонам уравнения: Y = 7
Дополнение: прямое решение квадратного уравнения
прямое решение y 2 -5y-14 = 0
Ранее мы разложили этот полином на множители, разделив средний член. давайте теперь решим уравнение, заполнив квадрат и используя квадратную формулу
Парабола, нахождение вершины :
3.1 Найдите вершину t = y 2 -5y-14
Параболы имеют самую высокую или самую низкую точку, называемую вершиной. Наша парабола раскрывается и, соответственно, имеет низшую точку (абсолютный минимум). Мы знаем это еще до того, как нанесем на график «t», потому что коэффициент первого члена, 1 , положителен (больше нуля).
Каждая парабола имеет вертикальную линию симметрии, проходящую через ее вершину. Из-за этой симметрии линия симметрии, например, будет проходить через середину двух точек пересечения x (корней или решений) параболы. То есть, если парабола действительно имеет два действительных решения.
Параболы могут моделировать многие реальные жизненные ситуации, такие как высота над землей объекта, брошенного вверх через некоторый период времени. Вершина параболы может предоставить нам такую информацию, как максимальная высота, на которую может подняться объект, брошенный вверх. По этой причине мы хотим иметь возможность найти координаты вершины.
Для любой параболы,Ay 2 +By+C, y -координата вершины определяется как -B/(2A) . В нашем случае координата y равна 2,5000
Подключение к формуле параболы 2.5000 для Y Мы можем рассчитать T -координату: T = 1,0 * 2,50 * 2,50 -5,0 * 2,50 -14,0 или T = -20,250
, график вершины и x -Intercepts:
Корневой график для: t = y 2 -5y-14 Ось симметрии (штриховая) {y}={ 2,50} Вершина в {y,t} = { 2,50,-20,25} y -Перехваты (корни ) : Корень 1 в точке {y,t} = {-2,00, 0,00} Корень 2 в точке {y,t} = {7,00, 0,00}
Решить квадратное уравнение, заполнив квадрат
3. 2 Решение y 2 -5y-14 = 0, заполнив квадрат .
Прибавьте 14 к обеим частям уравнения: y 2 -5y = 14
Теперь немного хитрости: возьмите коэффициент при y, равный 5, разделите на два, получив 5/2, и, наконец, возведите его в квадрат. что дает 25/4
Прибавьте 25/4 к обеим частям уравнения: В правой части мы получим: 14 + 25/4 или, (14/1)+(25/4) Общий знаменатель двух дробей равен 4 Сложение (56/4)+(25/4) дает 81/4 Таким образом, складывая обе части, мы окончательно получаем : y 2 -5y+(25/4) = 81 /4
Добавление 25/4 завершило левую часть в полный квадрат: y 2 -5y+(25/4) = (y-(5/2)) • (y-(5/2) )) = (y-(5/2)) 2 Вещи, равные одной и той же вещи, также равны друг другу. Так как y 2 -5y+(25/4) = 81/4 и y 2 -5y+(25/4) = (y-(5/2)) 2 тогда по закону транзитивности (y-(5/2)) 2 = 81 /4
Мы будем называть это уравнение уравнением. #3.2.1
Принцип квадратного корня гласит, что когда две вещи равны, их квадратные корни равны.
Обратите внимание, что квадратный корень из (y-(5/2)) 2 равен (y-(5/2)) 2/2 = (y-(5/2)) 1 = y-(5/2)
Теперь, применяя принцип квадратного корня к уравнению #3.2.1 получаем: y-(5/2) = √ 81/4
Добавьте 5/2 к обеим частям, чтобы получить: y = 5/2 + √ 81/4
другое отрицательное число y 2 — 5y — 14 = 0 имеет два решения: y = 5/2 + √ 81/4 или y = 5/2 — √ 81/4
8 1 √4. можно записать как √ 81 / √ 4 что равно 9/2
Решить квадратное уравнение, используя формулу квадратного уравнения
3.3 Решение y 2 -5y-14 = 0 по квадратичной формуле.
Согласно квадратичной формуле, y, решение для AY 2 +By +C = 0, где A, B и C цифры, часто называемые коэффициентами, определяются как: -B ± B B 2 -4AC y = ———————— 2A
В нашем случае A = 1 B = -5 C = -14
Соответственно, B 2 -4AC = 25-(-56) = 81
Применение квадратичной формулы:
5 ± √ 81 y = ———— 2
. упрощенный?
Да! Первичная факторизация числа 81 это 3•3•3•3 Чтобы иметь возможность удалить что-то из-под корня, должно быть 2 этих экземпляра (потому что мы берем квадрат, то есть второй корень).
Предполагается, что вы знакомы со следующим материалом, входящим в курс алгебры колледжа.
Пожалуйста, приведите себя в курс дела, если вы этого не сделаете.
Обозначение интервала
Альтернативной формой выражения неравенств является использование записи интервалов. Интервал
обозначение состоит из двух значений, разделенных запятыми.
Первое значение — это левая конечная точка интервала, а второе значение — правая. ручная конечная точка интервала. Левая конечная точка всегда находится слева, а
правая конечная точка всегда справа. Если неравенство продолжается вечно в
отрицательное направление, то левая конечная точка должна быть отрицательной бесконечностью. Если
неравенство всегда распространяется в положительном направлении, то правая конечная точка должна
быть положительной бесконечностью.
Каждая конечная точка заключена либо в квадратную скобку [ ], либо в круглую скобку ( ). Левая рука
конечной точке предшествует левая скобка [ или левая скобка ( и правая рука
за конечной точкой следует правая скобка ] или правая скобка ). Скобка означает, что
конечная точка включена, а скобки означают, что конечная точка не включена. Бесконечность (положительная или отрицательная) никогда не включается и всегда должна быть заключена в
скобка.
Открытый интервал — это когда обе конечные точки не включены ( ). Замкнутый интервал – это когда
обе конечные точки включены [ ]. Полуоткрытый (или полузакрытый, если вы пессимист)
интервал — это когда одна конечная точка включена, а другая конечная точка не является ( ] или [ ).
Примеры записи интервалов
Запишите: -2 < x < 5 как (-2, 5)
Запись: -2 <= x < 5 как [-2, 5)
Напишите: x >= 2 как [ 2, +∞ )
Запишите: x < 5 как (-∞, 5)
Абсолютное значение
Большинство людей знают, что абсолютное значение любого числа равно
нулевой или положительный. Однако математически это требует
кусочное определение.
При работе с абсолютными значениями это означает, что
удалить абсолютное значение из алгебраического выражения, выражение абсолютного значения
следует заменить двумя падежами. Один случай получается простым отбрасыванием абсолютного
знаки значения и уход от аргумента. Другой случай получается, если принять противоположное
аргумента функции абсолютного значения. Затем каждый случай прорабатывается
индивидуально.
Иногда можно решить уравнение, включающее абсолютные значения, с помощью
плюс-минус при исключении знаков абсолютного значения. Будьте осторожны при этом, и если вы
возникли трудности, затем вернитесь к двум отдельным случаям.
Еще один способ исключить абсолютное значение — возвести в квадрат обе части уравнения.
Принятие абсолютного значения делает вещи неотрицательными, а возведение в квадрат делает вещи неотрицательными. Итак, если вы что-то возвели в квадрат, вам больше не нужно брать его абсолютное значение.
Однако будьте осторожны при возведении в квадрат обеих частей уравнения, так как это может привести к
посторонние решения.
Свойства полей действительных чисел
Свойство замыкания — Сумма или произведение любых двух действительных чисел является другим действительным числом.
Коммутативное свойство — Порядок членов или факторов может быть изменен.
Ассоциативное свойство — Группировка терминов или факторов может быть изменена.
Идентификационные свойства — Ноль, добавленный к любому числу, является этим числом. Один умножить на любой
число это число.
Обратные свойства — Любое число плюс его противоположность равно нулю. Любое число, кроме нуля,
раз это взаимно один.
Распределительное свойство — Умножение распределяет по сложению.
Обратите внимание, что свойства определены для сложения и умножения. Некоторые из
свойства не работают для вычитания или деления.
Основная теорема арифметики
Каждое целое число больше единицы является либо простым числом, либо может быть записано как уникальное
произведение простых чисел.
Простые числа — ваши друзья. Уметь разлагать числа на простые множители, это сделает
жизнь намного легче потом.
Экспоненты
При умножении двух множителей с одинаковым основанием сложите показатели степени.
При умножении двух множителей с одинаковым показателем степени, но с разными основаниями умножьте
основания и сохранить показатель степени.
При возведении в степень умножьте степени вместе.
Научное обозначение
Уметь преобразовывать число из научной записи в обычную запись и из
обычная запись в научную запись.
Калькулятор TI-82/TI-83 использует клавишу EE в качестве клавиши экспоненциального представления. Когда
вы видите число, отображаемое как 1.253E12, что на самом деле означает 1.253×10 12 .
Корни
Знать, как преобразовать радикальную форму в рациональную экспоненциальную форму. В рациональном
показатель степени, знаменатель показателя степени является индексом корня и
числитель — мощность выражения.
Например, x 2/3 будет кубическим корнем x .2 .
Будьте осторожны, извлекая энный корень из энной степени. Если мощность четная, то вы
нужно брать абсолютное значение основания при упрощении радикала.
Упрощенная радикальная форма
Значение представляет собой упрощенную подкоренную форму, если выполняются следующие условия.
Показатели степени всех простых множителей подкоренного числа должны быть меньше
чем показатель радикала. По сути, это означает, что у вас не может быть квадрата.
корень х 3 .
В подкоренном члене нет дробей.
В знаменателе нет радикалов.
Нет общих множителей между показателями степени простого
множитель в подкоренном и индекс подкоренного. Это означает, что вы
следует уменьшить ваш индекс и мощность, если это возможно.
Факторные полиномы
Знать частные случаи разности двух квадратов, суммы двух квадратов
(что является простым числом над действительными числами и не учитывается), разница
из двух кубов, сумма двух
кубов, а разность двух n -й сил.
Разность двух квадратов: x 2 — y 2 = ( x — y ) ( x + y )
Сумма двух квадратов: x 2 + y 2 , не учитывает действительные числа
Разность двух кубов: x 3 — y 3 = ( x — y ) ( x 2 + xy + y 2 )
Сумма двух кубов: х 3 +
у 3 = ( х + у ) ( х 2 — ху + у 2 )
Разность двух n th градусов: x n — y n = ( x — y ) (x n-1 + x n-2 y + . .. + xy n- 2 + у п-1 )
Вышеприведенные шаблоны можно использовать в качестве рекомендаций. Например, 4x 2 -25 — это разность двух квадратов. Какая-то специально созданная сумма квадратов
будет фактором, но это
помимо того, что я ожидаю, что вы знаете для этого курса.
Знать, как разложить на множители трехчлен, не являющийся частным случаем. Вы можете найти
AC метод факторинга будет выгоден в этом случае.
Знать, как факторизовать по группировке.
Специальные продукты
Знать квадрат и куб двучлена.
Сумма в квадрате: ( x + y ) 2 =
х 2 + 2ху + у 2
Квадрат разности: ( x — y ) 2 = x 2 — 2xy + y 2
Сумма в кубе: ( x + y ) 3 = x 3 + 3x 2 y + 3xy 2 + y 3
Разность в кубе: ( x — y ) 3 = х 3 — 3х 2 у + 3ху 2 — у 3
Это частные случаи того, что известно как теорема о биномиальном разложении, которая будет рассмотрена в разделе
7. 5.
Дробные выражения
Выражения не имеют знаков равенства. Если бы это было так, то это были бы уравнения. Когда там
нет знака равенства, вы не можете умножить обе части уравнения (потому что нет
уравнения) к наименьшему общему знаменателю и исключить знаменатель.
Это означает, что при работе с рациональными (дробными) выражениями вы будете
иметь знаменатель в окончательном ответе (если только не произойдет деление с множителем в
числитель).
Вы делите или уменьшаете делителей на в числителе с делителями на в знаменателе. Делать
не отменять (если не хотите, чтобы кровь инструкторов закипела). Не разделяй
отдельные термины (факторы перемножаются вместе, термины складываются вместе).
Сложные дроби, содержащие одночлены, можно инвертировать, а затем умножить.
Однако, когда в составной дроби есть многочлены, обычно легче
умножьте верхнюю дробь и нижнюю дробь на наименьший общий знаменатель
два знаменателя.
Декартова плоскость
Уметь построить декартову систему координат. Это также известно как
x-y координатная плоскость. Знать названия квадрантов. Быть знакомым с
понятие упорядоченной пары
и
быть способным
заговор
точки
в системе
учитывая его координаты.
Координата x также известна как абцисса, а координата y также известна
как ордината.
Формулы
Вы должны знать следующие формулы.
Расстояние между двумя точками. Это в основном просто пифагорейский
Теорема. Найдите изменение координат x и изменение координат y.
Квадрат каждого из них и сложите их вместе. Наконец, извлеките квадратный корень.
Формула средней точки. Середина между двумя точками находится путем сложения
x и разделить на 2 и сложить Y и разделить на 2.
Уравнение окружности с центром (h,k) и радиусом r. (x-h) 2 + (y-k) 2 = r 2
Изучение данных
Вы не несете ответственности за этот раздел книги.
Об особом случае $x = -y$: он не такой уж особенный, но по технической причине, выходящей за рамки предварительного исчисления. Поскольку нетривиальные открытые множества Зарисского плотны в евклидовой топологии, а полиномиальные отображения непрерывны между евклидовыми топологиями, все неравенства вида $f(x_1, \ldots, x_n) \geq 0$ (где $f$ — многочлен ), которые выполняются на открытом множестве Зарисского, должны выполняться везде, потому что $[0, +\infty)$ замкнуто и его прообраз должен, таким образом, содержать замыкание указанного открытого множества Зарисского, т. е. все пространство. Следовательно, такие кажущиеся особыми случаи можно смело игнорировать, пока они закрыты по Зарисскому и неравенство не является строгим.
$\endgroup$ 2 $\begingroup$
Еще один — если $xy \le 0$ это тривиально, значит пусть $xy > 0$. Тогда мы имеем $$ 1 \le \dfrac{x}{y}+\dfrac{y}{x}$$
Теперь для любого положительного числа либо оно, либо его обратное значение должно превышать $1$, если только оба они не равны $1$ .
Excel для Microsoft 365 Excel для Microsoft 365 для Mac Excel для Интернета Excel 2021 Excel 2021 для Mac Excel 2019 Excel 2019 для Mac Excel 2016 Excel 2016 для Mac Excel 2013 Excel 2010 Excel 2007 Excel для Mac 2011 Excel Starter 2010 Еще…Меньше
В этой статье описаны синтаксис формулы и использование функции КОРЕНЬ в Microsoft Excel.
Описание
Возвращает положительное значение квадратного корня.
Синтаксис
КОРЕНЬ(число)
Аргументы функции КОРЕНЬ описаны ниже.
Замечание
Если число отрицательное, то SQRT возвращает #NUM! значение ошибки #ЗНАЧ!.
Пример
Скопируйте образец данных из следующей таблицы и вставьте их в ячейку A1 нового листа Excel. Чтобы отобразить результаты формул, выделите их и нажмите клавишу F2, а затем — клавишу ВВОД. При необходимости измените ширину столбцов, чтобы видеть все данные.
Данные
-16
Формула
Описание
Результат
=КОРЕНЬ(16)
Квадратный корень числа 16.
4
=КОРЕНЬ(A2)
Квадратный корень -16. Так как число отрицательное, #NUM! возвращается сообщение об ошибке.
#ЧИСЛО!
=КОРЕНЬ(ABS(A2))
Старайтесь не #NUM! сообщение об ошибке: сначала с помощью функции ABS можно найти абсолютное значение -16, а затем найти квадратный корень.
4
Как начертить график функции квадратного корня, (f(x)=√ x)
В этой статье будет показано, как набросать график функции квадратного корня, используя только три разных значения для ‘x’, а затем найти точки через который рисуется график Уравнений/Функций, также он покажет, как Графики Вертикально Перемещаются (движется вверх или вниз), Горизонтально Перемещаются (двигаются влево или вправо) и как График одновременно выполняет Оба Перевода.
Уравнение функции квадратного корня имеет форму… y = f(x) = A√x, где (A) не должно быть равно нулю (0). Если (A) больше нуля ( 0 ), то есть ( A ) является положительным числом, то форма графика функции квадратного корня аналогична верхней половине буквы «C». Если ( A ) меньше нуля ( 0 ), то есть ( A ) является отрицательным числом, форма графика аналогична форме нижней половины буквы «C». Пожалуйста, нажмите на изображение для лучшего просмотра.
Чтобы нарисовать график уравнения… y = f(x) = A√x, мы выбираем три значения для ‘x’, x = (-1), x = (0) и x = (1 ). Мы подставляем каждое значение ‘x’ в уравнение,… y = f(x) = A√x и получаем соответствующее значение для каждого ‘y’.
Учитывая y = f(x) = A√x, где (A) — действительное число и (A) не равно нулю (0), и подставляя x = (-1) в уравнение, мы получаем y = f(-1) = A√(-1) = i (это мнимое число). Итак, Первая точка не имеет реальных координат, следовательно, через эту точку нельзя провести график. Теперь подставив x = ( 0 ), мы получим y = f (0) = A√ (0) = A (0) = 0. Таким образом, вторая точка имеет координаты (0,0). И Подставляя x = ( 1 ), мы получаем y = f (1) = A√ (1) = A (1) = A. Таким образом, третья точка имеет координаты (1, A). Поскольку координаты первой Точки не были реальными, теперь мы ищем четвертую Точку и выбираем x = (2). Теперь подставьте x =(2) в y =f(2) = A√(2) = A(1.41)= 1.41A . Итак, четвертая точка имеет координаты (2,1.41A). Теперь мы нарисуем кривую через эти три точки. Пожалуйста, нажмите на изображение для лучшего просмотра.
Учитывая уравнение y = f(x) = A√x + B, где B — любое действительное число, график этого уравнения будет сдвигаться по вертикали ( B ) единиц. Если ( B ) является положительным числом, график будет двигаться вверх ( B ) единиц, а если ( B ) является отрицательным числом, график будет двигаться вниз ( B ) единиц. Чтобы нарисовать графики этого уравнения, мы следуем инструкциям и используем те же значения «x», что и в шаге № 3. Пожалуйста, нажмите на изображение, чтобы получить лучшее представление.
Учитывая уравнение y = f(x) = A√(x — B), где A и B — любые действительные числа, а ( A ) не равно нулю ( 0 ) и x ≥ B. График этого уравнения переведет по горизонтали ( B ) единиц. Если ( B ) является положительным числом, график будет двигаться вправо ( B ) единиц, а если ( B ) является отрицательным числом, график будет двигаться влево ( B ) единиц. Чтобы набросать графики этого уравнения, мы сначала устанавливаем выражение «x — B», которое находится под радикальным знаком больше или равно нулю, и находим «x». То есть… x — B ≥ 0, тогда x ≥ B.
Теперь мы будем использовать следующие три значения для ‘x’, x = (B), x = (B + 1) и x = (B + 2). Мы подставляем каждое значение ‘x’ в уравнение,… y = f(x) = A√(x — B) и получаем соответствующее значение для каждого ‘y’.
Учитывая y = f(x) = A√(x — B), где A и B — действительные числа, и ( A ) не равно нулю ( o ), где x ≥ B. Подставляя x = (B) в Уравнение мы получаем y = f(B) = A√(B-B) = A√(0) = A(0) = 0. Итак, первая точка имеет координаты (B,0). Теперь подставив x = ( B + 1 ), мы получим y = f (B + 1) = A√(B + 1 — B) = A√1 = A(1) = A. Таким образом, вторая точка имеет координаты ( B+1,A), и подставляя x = ( B + 2 ), получаем y = f(B+2) = A√(B+2-B) = A√(2) =A(1.41) = 1,41A . Итак, третья точка имеет координаты (B+2,1.41A). Теперь мы нарисуем кривую через эти три точки. Пожалуйста, нажмите на изображение для лучшего просмотра.
Дано y = f(x) = A√(x — B) + C, где A, B, C — действительные числа и ( A ) не равно нулю ( 0 ) и x ≥ B. Если C положительное Число, затем график в ШАГЕ № 7 будет перемещаться по вертикали ( C ) единиц. Если ( C ) является положительным числом, график будет двигаться вверх ( C ) единиц, а если ( C ) является отрицательным числом, график будет двигаться вниз ( C ) единиц. Чтобы набросать графики этого уравнения, мы следуем инструкциям и используем те же значения «x» из шага № 7. Пожалуйста, нажмите на изображение, чтобы получить лучшее представление.
Вещи, которые вам понадобятся
Бумага
Карандаш и
Миллиметровая бумага
Научитесь строить графики функций извлечения квадратного корня
В этом видео мы построим график функции извлечения квадратного корня. После того, как вы закончите этот урок, просмотрите все наши уроки по алгебре 1 и практические задачи.
Графический калькулятор является очень важным инструментом для построения графиков функции квадратного корня. Без калькулятора просто найдите квадрат каждого числа и нанесите точки на координатную плоскость. Соедините точки и не забудьте поставить стрелку, так как график продолжается. Имейте в виду, что квадратный корень из отрицательных чисел не существует.
Обратите внимание, что при вычитании числа из x под радикалом график сдвигается вправо на столько же единиц, а при добавлении числа к x под радикалом график сдвигается влево на столько же единиц. Кроме того, при добавлении числа вне корня график сдвигается вверх на столько же единиц, а при вычитании числа вне корня график сдвигается вниз на столько же единиц.
Примеры построения графика функций квадратного корня
Пример 1
Двигаемся влево на пробелы и вниз на . Наш график будет
Пример 2
Мы будем двигаться вправо на пробелы и вверх на . Наш график будет
Стенограмма видеоурока
Давайте рассмотрим, как построить график функции квадратного корня.
Начнем с самого простого.
Давайте посмотрим на это:
Если у нас есть , мы не можем получить квадратный корень из . Не существует. Это воображаемое.
Итак, если тогда для .
То же самое, когда .
Когда , , если , , если , , и если , .
Давайте просто выберем числа, из которых мы можем извлечь квадратный корень.
Используем , , , , , , , , и , .
Теперь построим график.
Это будет выглядеть так. Выровняется, но не полностью.
На другой стороне графика не было бы. Потому что квадратного корня из отрицательных чисел не существует.
Итак, это базовая форма функции извлечения квадратного корня.
Давайте более сложную функцию.
Например:
Вы должны помнить, что когда вы добавляете что-то к функции, строка поднимается на это количество пробелов.
В этом случае он поднимется на три позиции вверх.
А если что-то вычесть из , то мы сдвинемся вправо на это число.
Итак, в нашем примере мы будем двигаться вправо по пробелам.
Если у нас есть эта функция:
этот будет двигаться влево на пробелы и вниз на .
Давайте нарисуем эти два графика.
Линия будет двигаться в заданном нами направлении, но по-прежнему будет следовать той же кривой, что и основная функция.
Итак, если вы что-то добавите или вычтете из , оно сдвинется влево или вправо. Если вы прибавите или вычтете в конце, оно будет двигаться вверх или вниз.
При добавлении к , он перемещается влево. Затем, если вы вычитаете, он будет двигаться вправо.